Sie sind auf Seite 1von 76

Copyright © 2014 Delhi Academy of Medical Sciences, All Rights Reserved.

1/76
Test Information
Test Name SWT-OPTHALMOLOGY-2018 Total Questions 200

Test Type Examination Difficulty Level Difficult

Total Marks 800 Duration 120minutes

Test Question Language:- ENGLISH

(1). All of the following are true for the condition in the image except:

a. Results in circular areas of chorioretinal atrophy.

b. Is caused by ornithine aminotransferase deficiency

c. Is treated by arginine free diet

d. Is an autosomal dominant trait.

Solution. d
A missense mutation results in the deficiency of a specificenzyme, ornithineketoacidaminotransferase. the presenting symptom of
ornithine aminotransferase (OAT) deficiency is myopia which progresses to night blindness. Ophthalmological findings in affected
individuals include constricted visual fields, posterior subcapsular cataracts.
Ref – Kanski6thedi. Pg 693

Correct Answer. d

(2). In the early stages of Acanthmoeba keratitis, which of the following is least common?

a. In the early stages of Acanthmoeba keratitis, which of the following is least common?

b. Pseudodendrites.

c. Corneal neovascularization.

d. Infiltrates around corneal nerves.

Solution. c
Corneal neovasularization is uncommon. Early stages of AcanthamoebaKeratitis may be confined to the epithelium with irregularities,
infiltration or pseudodentrites. Perineuralinvasion results in prominent corneal nerves. Nonspecificstromalor characteristicring
infiltratesare seen as the diseases progresses.
Ref – Kanski6thedi. Pg 270

Correct Answer. c

Copyright © 2014 Delhi Academy of Medical Sciences, All Rights Reserved. 2/76
(3). What is true for episcleritis

a. Should be treated with tropical corticosteroids or nonsteroidal anti –inflammatory agents.

b. Usually (>50%) is associated with a systemic disease that should be treated.

c. Often needs systemic corticosteroids or nonsteroids.

d. Is almost always a self- limited condition that, if untreated, runs its course in a few days.

Solution. d
Episcleritisis the inflammation of the connective tissues betweenthe conjunctivaand sclera.It can present with either nodular or diffuse
inflammation. This self – limited condition typically resolves in a few days.Nodular episcleritis presents with more discomfort and takes
longer to resolve. No systemiccause is found in two – thirdsof the cases.
Ref – Kanski6thedi. Pg 324

Correct Answer. d

(4). The earliest sign of diabetic retinopathy seen clinically is:

a. Microaneurysms.

b. Dilated retinal venules (venous beadings).

c. Intraretinal hemorrhages.

d. Intraretinal exudates.

Solution. a
Microaneurysmsare the first ophthalmoscopicallydetectable change in diabetic retinopathy.
Ref – Wills eye Manual, 6th edi. Pg 295

Correct Answer. a

(5). The greatest use of indocyanine green angiography is to:

a. Detect chorodial neovascularization in presumed histoplasmosis.

b. Delineate the ischemic area in branch retinal artery occlusion.

c. Visualize cavernous hemangioma of the retina.

d. Detect occult, poorly defined chorodial neovascularization.

Solution. d
Indocyanine green (ICG)angiography is an infrared –based –imaging technique that is most useful for evaluating patients with exudative
changes from choroidal neovascularization(CNV).Particluarly, when occultCNVor poorly definedCNV by FA may be evaluated by ICG.
Ref – Textbook of Clinical Ophthalmology 3th edi. Pg 199

Correct Answer. d

(6). Artifacts of visual field testing than can produce results resembling true visual field defects include:

a. Cataracts.

b. Interference from lens holder rim.

c. Testing without correction of significant refractive errors.

d. Blepharoptosis.

Solution. a
Generalized depression of the visual field thatis caused by a cataract is not an artifact; it is a non glaucomatousvisual field defect.
Ref – Kanski6thedi. Pg 24

Correct Answer. a

Copyright © 2014 Delhi Academy of Medical Sciences, All Rights Reserved. 3/76
(7). False regarding the condition:

a. Associated with glaucoma usually involves both lids with portwine lesions.

b. Is associated with glaucoma in about one- third of patients.

c. associated with decreased episcleral pressure

d. With choroidal angimoa may be associated with subretinal hemorrhage when the globe is opened.

Solution. c
The cause of secondary glaucoma associated with SturgeWeber syndrome is not clear cut. If the angle is open,goniotomy
ortrabeculotomy are reasonable treatments. When the glaucoma is due to elevated episcleral venous pressure, atrabeculectomy or
drainage device may be a better option.
Ref – Textbook of Clinical Ophthalmology 3th edi. Pg 90

Correct Answer. c

(8). What is the angle subtended by the 6/60 letter on Snellen's chart, when read from 6 m

a. 5 min

b. 10 min

c. 25 min

d. 50 min

Solution. d
Minimal angle of resolution is reverse of VA. That is 60/6=10. It is for each arm of letter.
Each letter has 5 arms
So 10×5=50
Ref – Textbook of Clinical Ophthalmology 3th edi. Pg 497

Correct Answer. d

Copyright © 2014 Delhi Academy of Medical Sciences, All Rights Reserved. 4/76
(9). A person has fixed bulging right eye and unable to move in any direction with loss of sensations of cornea, cheek and upper face. What is
most likely the diagnosis

a. Orbital apex syndrome

b. Cavernous sinus syndrome

c. Myasthenia gravis

d. Superior orbital fissure syndrome

Solution. b
Cavernous Sinus Syndrome - III, IV, VI plus V1, V2, Horner’s syndrome
Superior orbital fissure- III, IV, VI plus V1
Orbital apex- III, IV, VI plus V1 and II
Cerebellopontine angle syndrome -V, VII, VIII CN palsies and cerebellar signs
Myasthenia gravis – Ophthalmoplegia, ptosis, pupils normal
Ref – Kanski6thedi. Pg 170

Correct Answer. b

(10). Which is false for right trochlear nerve nucleus

a. Nucleus is located in the mildbrain at the level of the inferior colliculus

b. Leaves the brainstem on its ventral aspect

c. Passes superior to the petroclinoid ligament

d. Supplies the left superior oblique

Solution. b
The level of trochlear nerve is at inferior colliculus (oculomotor nucleus is at level of superior colliculus) It is the only motor cranial nerve
to have origin from dorsal part of midbrain. The abducens nerve passes inferior to the petroclinoid ligament. It is the only motor cranial
nerve to decussate.
Ref – Wills eye Manual, 6thedi. Pg 247

Correct Answer. b

(11). Which of the following ocular structures have physiologically important cholinergic recptors?

a. extraocular muscles

b. ciliary body muscle

c. lacrimal gland

d. all of the above

Solution. d
Nicotinic cholinergic receptors mediate extraocular muscle contraction. Muscarinic receptors mediate automatic responses (e.g.,
accommodations and secretions).
Ref – Wills eye Manual, 6th edi. Pg 216

Correct Answer. d

Copyright © 2014 Delhi Academy of Medical Sciences, All Rights Reserved. 5/76
(12). The figure on the right shows stain of the canalicular reflux of a patient. What would you use to treat this patient?

a. surgical excision.

b. oral metronidazole.

c. oral amoxicillin / clavulanic acid.

d. intravenous ceftazidime.

Solution. c
The patient has canaliculitis, which is often caused by Actinomycesisraelli, a filamentous Gram – positive rod (as shown in the figure). It
is usually susceptible to penicillins and cephalosporins.
Ref – Kanski6thedi. Pg 162

Correct Answer. c

(13). Which of the following orbital tumors is least likely to present as a “masquerade syndrome

a. optic nerve glioma.

b. orbital dermoid cyst.

c. rhabdomyosarcoma.

d. acute leukemic orbital infiltration.

Solution. a
Each ofthe others may presentand be treated as orbital cellulitis. Metastaticneuroblastomaalso may simulate trauma, presenting with
spontaneous periocularecchymosis.
Ref – Wills eye Manual, 6th edi. Pg 360

Correct Answer. a

(14). The primary abnormality seen in ptosis after cataract surgery is in the;

a. levator muscle.

b. levator aponeurosis.

c. levator innervation

d. third nerve nucleus

Solution. b
Aponeuroticdehiscence has been blamed on anestheticinjection, lid specula, and bridle sutures. The exact cause is not clear.
Ref – Kanski6thedi. Pg 133

Correct Answer. b

Copyright © 2014 Delhi Academy of Medical Sciences, All Rights Reserved. 6/76
(15). After obtaining an applanation intraocular pressure (IOP) 42 mm Hg and attempting to examination the fundus unsuccessfully,
ultrasonography is performed and reveals a normal posterior segment. Gonioscopy reveals a completely closed angle. The next
intervention should be;

a. surgical revision of the wound

b. posterior sclerotomy

c. peripheral iridotomy

d. dilation with potent cycloplegics

Solution. c
Because pupillary block is probably present,aniridotomy should be performed. Miotics tend to increase postoperative inflammation and
should be avoided here.
Ref – Textbook of Clinical Ophthalmology 3th edi. Pg 61

Correct Answer. c

(16). Which of the following optic disc lesions are distinguished by autofluorescence?

a. myelinated nerve fibres

b. astrocytic hamartomas

c. optic nerve pits

d. optic nerve drusen.

Solution. d
Drusens have property of fluorescence, which is used in autofluoroscence.
Ref – Wills eye Manual, 6th edi. Pg 417

Correct Answer. d

Copyright © 2014 Delhi Academy of Medical Sciences, All Rights Reserved. 7/76
(17). Which of the following should probably be avoided in a patient with the condition shown in the OCT figure

a. topical beta – blockers

b. topical steroids

c. oral beta – blockers

d. oral steroids

Solution. d
Several case reports have documented significant deterioration in patients with central serous retinopathy (CSR) started or maintained
on oral corticosteroids. The most common finding is dramatically increased exudation with the development of bilateral bullous retinal
drtachments (RD). Fava beans should be avoided.
Email : info@damsdelhi.com | Website : www.damsdelhi.com Page | 3
in patients with glucose – 6 – phosphate dehydrogenase (G6PD) deficiency because they can precipitate acute hemolysis.
Ref – Textbook of Clinical Ophthalmology 3th edi. Pg 197

Correct Answer. d

(18). The only extraocular muscle which does not arise from the apex of the orbit is:

a. Superior rectus

b. Superior oblique

c. Inferior oblique

d. Inferior rectus

Solution. c
Rectus muscle originate from annulus of zinn Superior oblique from body of sphenoid Inferior oblique from body of maxilla
Ref – Kanski6thedi. Pg 735

Correct Answer. c

(19). What is the most common refractive error after Descemt’s membrane stripping in automated penetrating keratoplasty-

a. Hyperopia

b. Myopia

c. Simple myopic astigmatism

d. Simple hyperopic astigmatism

Solution. a
Hyperopia of 1-1.5 D is induced after DSAEK
DSAEK is a procedure which involves stripping of the diseased endothelium and descements membrane with the donor graft which
comprises of endothelium, descements membrane and a small strip of posterior stroma. This donor graft is harvested with a automated
keratome.Since no sutures are required the rehabilitation time is less compared to conventional penetrating keratoplasty[PK]. The
endothelial loss is more compared to PK and ranges from 30 – 40%
Ref – Textbook of Clinical Ophthalmology 3th edi. Pg 151

Correct Answer. a

Copyright © 2014 Delhi Academy of Medical Sciences, All Rights Reserved. 8/76
(20). An infant presents with posterior lenticonus, cerebral and renal disorders with presence of amino acids in urine. All of the following are
characteristics of this condition EXCEPT:

a. autosomal dominant inheritance

b. renal tubular acidosis

c. bilateral congenital cataracts

d. Infantile glaucoma

Solution. a
Lowe’s syndrome is an X-linked recessive disorder characterized by renal tubular acidosis, bilateral congenital cataracts, glaucoma,
mental retardation, muscular hypotonia, and failure to thrive.
Ref - Glaucoma Medical Diagnosis and Therapy, 2ndedi, pg 398

Correct Answer. a

(21). A hypermetropic patient with closed angles on gonioscopy has IOP of 25 mm Hg with normal optic disc and visual fields. Diagnosis?

a. Primary angle closure suspect

b. Primary angle closure

c. Primary angle closure glaucoma

d. Ocular hypertension

Solution. b

Correct Answer. b

Copyright © 2014 Delhi Academy of Medical Sciences, All Rights Reserved. 9/76
(22). An 89 – years – old aphakic patient is scheduled for a secondary lens implant. All the following options are viable except:

a. Sulcus placement

b. Anterior chamber placement

c. Trancssleral sutured posterior chamber placement

d. Iris – sutured posterior chamber placement

Solution. a
Placing an IOL in the sulcus in an aphakic eye will not work because of the lack of posterior support and the probability that the IOL will
end up in the vitreous compartment.
Ref – Textbook of Clinical Ophthalmology 3th edi. Pg 41

Correct Answer. a

(23). All but one of the following are true for the condition in the figure except

a. there is a gentic link to the gene LOXL1 on chromosome 15

b. The peak age is 45.

c. It is more often in Scandinavian countries

d. There are transillumination defect on the pupilary borders

Solution. b
PSEUDO-EXFOLIATIVE GLAUCOMA - (GLAUCOMA CAPSULARE) - Due to deposition of grey dandruff- like material in the trabecular
meshwork. It is unusual for it to be diagnosed before the age of 50 years.
Ref – Kanski6thedi. Pg 397

Correct Answer. b

Copyright © 2014 Delhi Academy of Medical Sciences, All Rights Reserved. 10/76
(24). Which is false for Sympathetic Uvetits:

a. Requires presence of a penetrating sclera wound.

b. Is associated with a cell –mediated immune response to retinal antigen.

c. May be associated with papillitis.

d. Is more common in middle aged

Solution. d
Sympathetic uveitis is more common in2 agegroups:childrenunderthe age of 10 years (due to ocular injuries) and over the age of 60
years (due to surgical interventions).
• Very rare, bilateral, granulomatous panuveitis.
• Occurs after accidental penetrating trauma.
• Traumatized eye is referred as the exciting eye and the fellow eye is sympathizing eye.
• 65% of cases occur between 2 wks and 3 months after injury
• 90% of cases occur within first year
Ref – Textbook of Clinical Ophthalmology 3th edi. Pg 421

Correct Answer. d

(25). Nonarteritic anterior ischemic optic neuropathy is associated with:

a. Systemic hypertension.

b. Diabetic mellitus.

c. Carotid occlusive disease

d. Magnetic resonance imaging evidence of cerebral small vessel ischemic disease.

Solution. c
Non-Arteritic AION:
Etiology: Unknown but the major risk factor is Hypertension.
Clinical features:
1. Sudden painless loss of vision
2. Altitudinal field defect,commonly involving the inferior field.
3. No premonitory visual symptoms
4. Slightly decreased visual acuity. Dimunition of color vision is in proportion to the decrease of visual acuity.
5. Sectoral or Diffuse edema of the disc. Optic nerve head is either pale or hyperaemic.
6. Flame –shaped haemorrhages around the disc.
Ref – Kanski6thedi. Pg 789

Correct Answer. c

(26). True regarding vitreous base is:

a. Shows traumatic retinal breaks as its posterior aspect.

b. Shows rhegmatogenous retinal breaks as its anterior aspect.

c. Is more prominent in the lightly pigmented fundus, an area where the vitreous body, neural retina, and pigment epithelium all are
loosely adherent to one another.

d. Measures about 3. 2 mm in width,

Solution. d
The vitreous base involves the full circumference of the peripheral fundus and measures about 3.2 mm in width. In terms of retinal
breaks, they frequentlyoccur along the posterior border, in the case of traumatic detachment occasionally along the anteriorborder as
well. The description of posterior and anterior aspect is not accurate.
Ref – Textbook of Clinical Ophthalmology 3th edi. Pg 507

Correct Answer. d

Copyright © 2014 Delhi Academy of Medical Sciences, All Rights Reserved. 11/76
(27). Examination of a young male, as seen in the figure revealed fine keratic precipitates, few cells in anterior chamber. Which of the
following are true regarding the condition except

a. Occurs with equal frequency in men and women.

b. May appear at any sge

c. Has been associated with immune response to S-antigen

d. Is usually bilateral.

Solution. d
Unilateral anterior uveitis is a typical presentation of Fuch’sheterochromiciridocyclitis.
It is an insidious, non-granulomatous, anterior uveitis affecting one eye of a middle aged patient
Clinical features:
a. Uniocular blurring of vision secondary to cataract formation.
b. Keratic precipitates- are pathognomic as they are small, stellateand grey-coloured; scattered throughout the cornea. These KPs never
become confluent or pigmented. Feathery fibrin filaments may be seen in between the KPs.
c. Aqueous humour:Faint flare and New cells
d. Absence of posterior synechiae
e. Iris stromal atrophy: May be associated with patchy atrophy of posterior pigment layer of iris.
f. Heterochromia iridis
Ref – Kanski6thedi. Pg404

Correct Answer. d

(28). Which of the following options is the most appropriate management plan for para planitis?

a. topical steroids

b. sub-Tenon steroid injection

c. oral methotrexate administration

d. argon laser treatment of the peripheral retina

Solution. b
Sub-Tenon steroid injection is a local treatment that is effective in managing inflammation and macular edema associated with pars
planitis, especially when only 1 eye is involved. Immunomodulation is reserved for persistent or refractory cases. Argon laser treatment
is utilized for peripheral retinal neovascularization associated with the disease. Pars plana vitrectomy is reserved for serious
complications such as retinal detachment, advanced uveitis, cataract, and/or severe cystoid macular edema (CME) unresponsive to
corticosteroid therapy.
Ref – Textbook of Clinical Ophthalmology 3thedi. Pg 417

Correct Answer. b

Copyright © 2014 Delhi Academy of Medical Sciences, All Rights Reserved. 12/76
(29). Which of the following is a neural crest derivative

a. Lens

b. Extraocular muscles

c. Trabecular meshwork

d. Sphicter pupillae

Solution. c
Neural Crest derivatives are sclera, corneal stroma and endothelium, Schlem’s canal, trabecular meshwork, ciliarybody muscles
Neuroepithelium / Optic cup / Neuroectodermforms retina, iris and ciliary body epithelium, dilator and sphincter puppilae, seconday and
tertiary vitreous
Surface ectoderm forms corneal and conjunctival epithelium and glands derived from them, lens, lacrimal gland
Mesoderm forms extraocular muscles, blood vessels, primary vitreous

Correct Answer. c

(30). A 50 yr old male comes to clinic with right eye proptosis, total external ophthalmoplegia and loss of corneal sensations. The sensations
over cheek area were normal. On investigation, no cause can be determined. What is this condition?

a. Tolosa Hunt syndrome

b. Cavernous sinus syndrome

c. Superior orbit fissure syndrome

d. Orbit apex syndrome

Solution. a
It is due to an idiopathic orbital apex inflammatory mass. The pain is due to fifth cranial nerve involvement. There is usually an early and
beneficial response with steroids. Due to involvement of cranial nerves at the orbital apex, opthalmoplegia is there.
Ref – Wills eye Manual, 6thedi. Pg 155

Correct Answer. a

Copyright © 2014 Delhi Academy of Medical Sciences, All Rights Reserved. 13/76
(31). In which layer is this sign located in

a. nerve fibre layer.

b. ganglion cell layer

c. inner plexiform layer.

d. inner nuclear layer

Solution. a
Nerve fiber Layer
It is due to infarction of nerve fibre layer
Ref – Kanski6thedi. Pg568

Correct Answer. a

(32). A patient with known neurofibromatosis presents with pulsating proptosis of long duration. Computed tomography (CT) scan of the orbit
will most likely reveal;

a. orbital neurofibroma

b. cavernous hemangioma

c. abnormality of the sphenoid bone

d. optic nerve glioma

Solution. c
Dysplasia or aplasia of the sphenoid bone creates large posterior orbital defects. This leads to pulsatingexopthalmos, as brain
tissueherniates outwards.
Ref – Kanski6thedi. Pg 189

Correct Answer. c

Copyright © 2014 Delhi Academy of Medical Sciences, All Rights Reserved. 14/76
(33). A 54 – year – old man presents with intraocular pressures (IOP) of 24 mm Hg in the the right eye and 26 mm Hg in the left eye. His visual
fields and optic nerve heads appear normal. Which of the following clinical features might prophylactic topical medical therapy except?

a. elevated pachymetry measurements

b. family history of glaucoma

c. decreased nerve fiber layer thickness measurements using optical coherence tomography

d. African American descent

Solution. a
Treatment of the suspected patient with glaucoma should be limited to patients with a high risk of ultimate damage to the optic nerve
caused by elevated intraocular pressure (IOP). These risk factor include elevated IOP, positive family history of glaucoma, myopia,
diabetes mellitus, cardiovascular disease, race (i.e., African American), asymmetric cupping, large cups, and early nonspecific visual field
changes. The Ocular HypertensionTreatment Study has demonstrated that individuals with ocular hypertension and thin corneas are
most likely to develop glaucoma.Decreased nervefiberlayer thickness readings using optical coherence tomography (OCT) would make
one suspicious for optic nerve damage.
Ref – Textbook of Clinical Ophthalmology 3tthedi. Pg 71

Correct Answer. a

(34). Which one of the following statements about optic nerve drusen is false?

a. Optic nerve drusen are generally silent clinically.

b. They are generally bilateral

c. They are common in Africans Americans and Asians

d. Associated visual field defects may resemble those of glaucoma

Solution. c
Optic nerve drusen occur almost exclusively in white individuals. Ref – Wills eye Manual, 6th edi. Pg 395

Correct Answer. c

(35). To quantify a hyperdeviation accurately, a prism is most appropriately placed in front of the deviated eye;

a. base – up

b. base –out

c. base – down

d. base – in

Solution. c
The base of prism is always placed opposite to the direction of the deviation. Ref –Kanski6thedi. Pg 748

Correct Answer. c

Copyright © 2014 Delhi Academy of Medical Sciences, All Rights Reserved. 15/76
(36). A 30 years old male presents with a history of injury to the eye with a leaf 5 days ago and pain, photophobia and redness of the eye for 2
days. The figure is given below What would be the most likely pathology?

a. Anterior uveitis

b. Conjunctivitis

c. Fungal corneal ulcer

d. Corneal laceration

Solution. c
Fungal keratitis
Clinical Features
1. Watering, photophobia, redness, pain, mild discharge
2. Fungal ulcer – dry and rough
3. Stromal infilterates with feathery hyphate edges
4. Wessely ring – around central lesion
5. Satellite lesions in periphery
6. Associated hypopyon: Unsterile
Ref–Ophthalmologyyanoffand duker4/ep 225

Correct Answer. c

(37). Which one of the following constitutes a violation of Sherrington’s law?

a. Inhibitional palsy of the contralateral antagonist

b. Disassociated vertical divergence

c. Brown’s syndrome

d. Duane’s syndrome

Solution. d
In Duane’s syndrome, innervational impulses to the medial rectus are not associated with decreased innervation of the ipsilateral lateral
rectus muscle, as Sherrington’s law demands. This is because of anomalous innervations of the lateral rectus, in this case from the
oculomotor (III) nerve, which leads to coinnervation of the horizontal rectus muscle, with globe retraction on adduction.

Correct Answer. d

Copyright © 2014 Delhi Academy of Medical Sciences, All Rights Reserved. 16/76
(38). While undergoing retinal examination, a patient claim to see his retinal vessels. This is an example of

a. formed hallucinations

b. Charles Bonnets syndrome

c. blindsight.

d. Purkinje effect

Solution. d
The Purkinje effect is a benign condition. It is a real, subjective test and can rarely be used to determine whether a patient’s retina is
attached.
Ref – Textbook of Clinical Ophthalmology 3tthedi. Pg 177

Correct Answer. d

(39). A circular, superficial distribution of neutrophils around an area of corneal edema or inflammation is called;

a. Wessely’s

b. Disciform keratitis

c. Corneal abscess

d. metaherpetic ulcer

Solution. a
The phenomenon is identical to the immunoprecipitate formed in the Ouchterlony gel.
Ref – Kanski6thedi. Pg 270

Correct Answer. a

(40). A 73 – year – old man reports to the office on the first day following his cataract extraction complaining of severe eye pain. Visual acuity
is counting fingers at 3 feet in the involved eye. Slit – lamp examination reveals a diffusely shallow anterior chamber and corneal edema
without hypopyon. The next step should be;

a. dilated fundus examination

b. tonometry

c. gonioscopy

d. pachymetry

Solution. b
High intraocular pressure (IOP) secondary to angle closure may be causing severe eye pain and decreased visual acuity. Hypotony
(wound leak,cyclodialysis) could precipitate painful choroidal heamorrhage.
Ref – Wills eye Manual, 6th edi. Pg 416

Correct Answer. b

Copyright © 2014 Delhi Academy of Medical Sciences, All Rights Reserved. 17/76
(41). Which one of the following clinical findings at presentation is inconsistent with the diagnosis in a 5year old female as in the figure?

a. insidious onset

b. afferent pupillary defect (APD)

c. pain.

d. axial proptosis

Solution. c
Pain is quite unusual.
There is Fusiform enlargement of ON (usually seen associated with neurofibromatosis)
• Arises from glial tissue (astrocytes, oligodendrocytes, ependymal cells)
• Young girls (2–6 years)
• Axial proptosis occurs early
• VA decrease occurs early
• Optociliary shunt uncommon
• Associated with Type Ineurofibromatosis in 20–49% (keyword)
Ref – Textbook of Clinical Ophthalmology 3th edi. Pg 377

Correct Answer. c

(42). Compared with plasma, aqueous humour has an increased concentration of which one of these components?

a. Protein

b. Ascorbate

c. Glucose

d. Carbon dioxide

Solution. b
Compared with plasma, aqueous is slightly hypertonic and acidic. Aqueous has a marked excess of ascorbate (15 times greater than that
of arterial plasma) and a marked deficit of protein (0.2% in aqueous as compared to 7% in plasma).
Ref - Glaucoma Medical Diagnosis and Therapy, 2nd⁄ edi, pg 40

Correct Answer. b

Copyright © 2014 Delhi Academy of Medical Sciences, All Rights Reserved. 18/76
(43). What is the percentage of trypan blue dye used in preparation for creating a capsulorrhexis during cataract surgery

a. 1%

b. 0. 6%

c. 0. 06%

d. 0. 006%

Solution. c.
Trypan blue is a nontoxic and commercially available as a ready made solution.
Rose Bengal is not approved for intraocular use and has been shown to be toxic to the corneal endothelial cells in animal studies.
Ref – Textbook of Clinical Ophthalmology 3th edi. Pg 15

Correct Answer. c

(44). What is the most likely diagnosis in a 14 year old girl complaining of poor central vision but no abnormalities on dilated fundus
examination. There is no family history of such condition

a. central serous chorioretinopathy

b. Sjogren-Larsson syndrome

c. Best's disease

d. Stargardt's disease

Solution. d
The age of presentation and normal fundus appearance suggest Stargardt's disease.
Stargardt’s (fundus flavimaculatus) is characterised by: • most common inherited macular dystrophy
• autosomal recessive
• presents in first 2 decades
• bilateral reduced vision
• fundus may appear normal
• early: mottling of RPE; pisciform flecks at the level of the RPE at the posterior pole
• later: aggregation of flecks to form an atrophic patch and bull's eye appearance
• FFA: dark or 'silent choroid' due to blockage of the background choroidal fluorescence by deposition of lipofuscin
• Visual prognosis is generally poor with vision stabilizing to 6/60
Ref – Textbook of Clinical Ophthalmology 3thedi. Pg 519

Correct Answer. d

(45). Exudative detachments occur in which of the following conditions:

a. Vogt-Koyanagi-Harada Syndrome (VKH)

b. myopia

c. Sickle cell retinopathy

d. Diabetic retinopathy

Solution. a
Vogt-Koyanagi-Harada Syndrome (VKH)has multiple exudative retinal detachments Myopia is associated with rhegmatogenousretinal
detachments. Sickle cell retinopathy and diabetic retinopathy are associated with tractional detachment.
Ref - Alberts Principles and Practice of Ophthalmology, 3rdedi – pg 2305

Correct Answer. a

Copyright © 2014 Delhi Academy of Medical Sciences, All Rights Reserved. 19/76
(46). The “ tomato ketchup” fundus is a classic finding in which of the phakomatoses?

a. neurofibromatosis.

b. Sturge- Weber syndrome

c. tuberous sclerosis.

d. ataxia-telangiectasia.

Solution. b
Sturge – Weber syndrome is a sporadic diseaseassociated with a distinctive, diffuse chorodialhermangioma, whichcauses a reddish,
thickened appearance oflarge areas of the fundus that has been likened to tomata ketchup. Glaucoma and retinal detachment can occur
in eyes with a tomato ketchup fundus.
Ref – Textbook of Clinical Ophthalmology 3thedi. Pg 507

Correct Answer. b

(47). Which of the following lenticular diagnoses is demonstrated in the figure below?

a. Cogan – Reese syndrome.

b. Mittendorf’s dot.

c. Ectopia lentis

d. Soemmerring’s ring

Solution. d
A soemmerring’s ring is shown, representing incomplete removal of cortex and /m or nuclear fibers leading to a doughnut – shaped
configuration of the lens.
Ref – Textbook of Clinical Ophthalmology 3th edi. Pg 507

Correct Answer. d

(48). Which of the following gonioscopic methods is / are direct?

a. Goldmann

b. Richardson

c. Zeiss

d. Koeppe

Solution. c
Koeppe and Richardson lenses are used in direct gonioscopy, whereas the Goldmann and Zeiss lenses are used in indirect goniscopy.
Direct gonioscopy is useful during intraocular surgery (e.g.,goniotomy).
Ref – Wills eye Manual, 6th edi. Pg 417

Correct Answer. c

Copyright © 2014 Delhi Academy of Medical Sciences, All Rights Reserved. 20/76
(49). The most common cause of acquired fourth nerve palsy in adults is;

a. tumor.

b. trauma.

c. vascular disorder

d. idiopathic palsy

Solution. b
The long intracranial course of the trochlear nerve leaves it especially susceptible to damage from closed head trauma. This occurs
because of contrecoup injury from the free tentorial edge. Ischemic damage, usually caused by diaetes mellitus, is second, and idiopathic
palsies are third. Hydrocephalus, vascular loops, or tumor can compress the trochlear nerve as well.
Ref – Kanski6thedi. Pg 821

Correct Answer. b

(50). Which of the following statements about the condition shown in the figure is false?

a. the condition is unilateral.

b. Retinal vessels originating from the periphery of the disc is a classic finding

c. Patients with this condition are at risk for retinal detachments

d. Visual acuity is often normal

Solution. d
The fundus photograph demonstrates a morning glory disc anomaly, a unilateral condition usually accompanied with severe visual loss. It
can also cause relative afferent papillary defect (RAPD). The other ansewers are correct.
Ref – Wills eye Manual, 6th edi. Pg 245

Correct Answer. d

Copyright © 2014 Delhi Academy of Medical Sciences, All Rights Reserved. 21/76
(51). The systemic evaluation of a patient with biopsy – proven orbital lymphoma generally includes all of the following except;

a. bone scan

b. bone marrow biopsy

c. liver and spleen scan.

d. lumbar puncture

Solution. d
The central nervous system (CNS) is not routinely surveyed in patientswith orbital lymphoma. This is in contrast to patients with
intraocular lymphoma. When (CNS) involvement is suspected, computed tomography(CT) scan or magnetic resonanceimaging (MRI)is
the starting point.
Ref – Kanski6thedi. Pg190

Correct Answer. d

(52). Which is the only muscle supplied by the oculomotor nerve that receives crossed innervation?

a. superior rectus.

b. medial rectus

c. inferior oblique

d. levator palpebrae superioris

Solution. A
The superior recti have two subnuclei, each controlling the contralateral nerve.
Ref – Textbook of Clinical Ophthalmology 3tth edi. Pg 507

Correct Answer. a

(53). Which statement is FALSE regarding the lacrimal gland:

a. The orbital and palpebral portions are connected via an isthmus

b. destruction of the palpebral portion removes 50-70% of tear production

c. the orbital portion is larger than the palpebral portion

d. the orbital portion is lodged in the fossa for lacrimal gland

Solution. b
The lacrimal gland is composed of two portions: a larger orbital portion and a smaller palpebral portion (about one-quarter the size). The
three to five main excretory ducts of the orbital portion must traverse the palpebral portion, which contributes a further five to seven
ducts to the total of about 12 excretory ducts. As all ducts pass through it, destruction of the palpebral portion renders the entire gland
non-functional.
Ref – Wills eye Manual, 6thedi. Pg 144

Correct Answer. b

Copyright © 2014 Delhi Academy of Medical Sciences, All Rights Reserved. 22/76
(54). A 60 –year –old women with history of a primary cancer (surgical removal within the prior 5 months) complains of blurred vision. A
golden yellow lesion, presumably metastatic, is noted in the choroid. The most likely primary site is the:

a. Cervix.

b. Kidney.

c. Lung.

d. Ovary.

Solution. c
Breast and lungs are far and away the two most frequent primary sites of metastatic carcinoma to the eye in women
Ref – Kanski6thedi. Pg190

Correct Answer. c

(55). All of the following are associated with this condition except:

a. Aniridia

b. Sulphite oxidase deficiency

c. Diabetes mellitus

d. Hyperlysinemia

Solution. c
Ectopialentis is associated with:
• Familial ectopialentis
• Marfan syndrome
• Weill-Marchesani
• Homocystinuria
• Familial microspherophakia
• Hyperlysinaemia
• Sulphite oxidase deficiency
• Stickler syndrome
• Sturge-Weber syndrome
• Crouzon syndrome
• Ehlers-Danlos syndrome
• Aniridia
Ref – Kanski6thedi. Pg368

Correct Answer. c

Copyright © 2014 Delhi Academy of Medical Sciences, All Rights Reserved. 23/76
(56). All of the following are characteristics of 3rd nerve palsy EXCEPT:

a. esodeviation

b. pupil dilatation

c. hypotropia

d. ptosis

Solution. a
Third nerve palsy may result in complete or partial loss of superior, medial, and inferior rectus function, as well as inferior oblique and
levator function. The eye is usually deviated down and out. Aberrant reinnervation may manifest as abnormal pupillary constriction with
adduction.
Ref – Kanski6thedi. Pg 816

Correct Answer. a

(57). Which cell is thought to be the most closely related to these cells?

a. Ganglion cell

b. Photoreceptor

c. Retinal pigment epithelium

d. Muller cell

Solution. b
Figure shows a Flexner-Wintersteiner rosette, characteristic of retinoblastoma. These rosettes represent abnormal photoreceptor
differentiation. Homer-Wright rosettes and fleurettes also along the spectrum of photoreceptor differentiation.
Ref – Kanski, 6thedi, pg 542

Correct Answer. b

(58). Which of the following is / are variations of the hyaloidolenticular vascular system?

a. Bergmeister’s papilla

b. persistent hyaloid artery.

c. Mittendorf’s dot.

d. all

Solution. d
Bergmeister’s papilla is a nonpatent hyaloids artery remnant extending from the optic disc. If this structure is patent, it is called
persistent hyaloid artery.Mittendorf’s dot is an opacity on the inferonasal posterior lens capsule from connective tissues associated with
the centralhyaloid vasculature and posterior tunica vasculosa.

Correct Answer. d

Copyright © 2014 Delhi Academy of Medical Sciences, All Rights Reserved. 24/76
(59). Dalen – Fuchs nodules may be seen in which of the following conditions except?

a. Sympathetic ophthalmia (SO)

b. tuberculous choroiditis

c. Vogt – Koyanagi – Harada (VKH) syndrome

d. Behcet’s disease

Solution. d
Dalen – Fuchs nodules are focal accumulations of epithelioidlike cells between Bruch’s membrane and the retinal pigment epithelium
(RPE). They may include depigmented RPE cells.They are classically associated with sympathetic ophthalmia(SO) and Vogt – Koyanagi –
Harada (VKH) syndrome. They also may be found in tuberculous choroiditis and sarcoidosis.
Ref – Kanski6thedi. Pg 466

Correct Answer. d

(60). Which of the following features is / are consistent with a temporal lobe lesion?

a. seizures.

b. optokinetic nystagmus (OKN) abnormalities.

c. formed visual hallucinations.

d. high congruity of visual field deficits.

Solution. -NA-

Correct Answer. b

(61). A CT scan was obtained and is shown in figure. What is the probable diagnosis?

a. Retinoblastoma

b. Leukemia

c. Lymphoma

d. Choroidal melanoma

Solution. a
The CT scan shows bilateral high density lesions consistent with calcification within the tumors, seen in retinoblastoma
Ref – Kanski6thedi. Pg 542

Correct Answer. a

Copyright © 2014 Delhi Academy of Medical Sciences, All Rights Reserved. 25/76
(62). All of the following are used to evaluate the optic disc except:

a. direct opthalmoscopy

b. Slit-lamp examination using a contact lens

c. Indirect ophthalmoscopy using a 20-D lens

d. Slit-lamp examination using 90-D lens

Solution. c
The small image obtained using indirect opthalmoscopy does not allow for adequate evaluation of the optic nerve details. Careful
examination with the direct opthalmoscope can provide important information about the pallor of the optic cup; however, the most
effective methods include stereoscopic examination using the slit lamp in combination with a posterior-pole contact lens, a 90-D lens, or
a Hruby lens.
Ref - Glaucoma Medical Diagnosis and Therapy, 2ndedi, pg202

Correct Answer. c

(63). Which one of the following structures does not insert into the lateral orbital tubercle of Whitnall?

a. check ligament of the lateral rectus

b. suspensory ligament of the eyeball (Lockwood’s ligamemt)

c. lateral canthal tendon

d. Whitnall’s ligament.

Solution. d
The lateral orbital tubercle of whitnall is a small elevation in the orbital margin ofthe zygoma. It lies 11 mm below the frontal
zygomaticsuture. The tubercle isan important attachment site for all of the structures listed in the question except (ironcally) Whitnall “s
ligament. This structure is a condensation of fascia in the superior orbit that inserts 10 mm above Whitnall”s tubercle.
Ref – Kanski6thedi. Pg 166

Correct Answer. d

(64). What is the most common intraocular infection in patients with AIDS?

a. acute retinal necrosis

b. toxoplasmosis

c. candidiasis

d. cmv retinitis

Solution. d
Cytomegalovirus retinitis remains the most common intraocular infection in patientswith AIDS, despite a decline in the number of new
AIDS cases.
Ref - Alberts Principles and Practice of Ophthalmology, 3rdedi – pg 2107

Correct Answer. d

Copyright © 2014 Delhi Academy of Medical Sciences, All Rights Reserved. 26/76
(65). How many bones compose the lacrimal sac fossa?

a. 1

b. 2

c. 3

d. 4

Solution. b
The lacrimal sac fossa is formedby the frontal process of the maxilla and the lacrimal bone. It cradles the lacrimal sac and is continuous
with the nasolacrimalcanal
Ref –Kanski6thedi. Pg 166

Correct Answer. b

(66). Which is false for the following condition

a. Is always bilateral.

b. Also consists of telecanthus and epicanthus inversus.

c. Is commonly associated with mental retardation

d. Is dominantly inherited.

Solution. c
Blepharophimosissyndrome is not commonly associated with mental retardation. It is associated with primary amenorrhea in some family
lines.
Ref – Textbook of Clinical Ophthalmology 3th edi. Pg 507

Correct Answer. c

(67). Which of the following is not associated with cataract formation?

a. Radiation

b. Retinitis pigmentosa

c. Waardenburg’s Syndrome

d. Lowe’s syndrome

Solution. c
Radiation, Lowe’ssyndrome, and retinitis pigmentosaareoftenassociated with cataract. Waardenburg’s syndromeis not.
Ref – Textbook of Clinical Ophthalmology 3th edi. Pg 507

Correct Answer. c

Copyright © 2014 Delhi Academy of Medical Sciences, All Rights Reserved. 27/76
(68). All the following are located in the inner nuclear layer of the retina except:

a. Amacrine cell nuclei

b. Horizontal cell nuclei.

c. Muller cell nuclei

d. Rod nuclei.

Solution. d
Muller cells, bipolar cells, and the horizontalandamacrine cells are located in the inner nuclear layer.
Ref – Textbook of Clinical Ophthalmology 3th edi. Pg 507

Correct Answer. d

(69). Fluorescein angiography is useful for all conditions except

a. Chorodial artery thrombosis.

b. Diabetic retinopathy

c. Cystoids macular edema

d. Central serous chorioretinopathy

Solution. a
Although fluorescein angiography (FA) has contributed greatly to the diagnosis of many common choriorental diseases, limitations
occurto FA, mostly the ability to image the choroidal circulation.
Ref – Wills eye Manual, 6th edi. Pg 295

Correct Answer. a

(70). Which is false for the instrument in the figure?

a. Acts as a line of pinholes

b. Can be used to screen for astigmastim

c. Is most useful in patients who have better than 6 / 12 visual acuity.

d. Is found in most trial lens sets

Solution. c
The stenopaeic slit is a black disk with a thin slit through it used in detecting the level of an astigmatism. The disk is rotated so that the
slit is oriented at different angles, changing the retinal blur size. The stenopeic slit is more useful to screen for a high degree of
astigmatism n patients who have poor vision.
Ref – Kanski6thedi. Pg 746

Correct Answer. c

Copyright © 2014 Delhi Academy of Medical Sciences, All Rights Reserved. 28/76
(71). Organisms commonly associated with late-onset postoperative endopthalmitis include:

a. Staphylococcus epidermidis.

b. Propionbacterium acnes

c. Candida parapsilosis

d. Staphylococcus aureus

Solution. b
Late onset endophthalmitis is chronic and mild condition, usulayyocuuring after 1 month of cataract surgery
Ref – Kanski6thedi. Pg 339

Correct Answer. b

(72). A patient has an anterior chamber intraocular lens placed into the eye after a complicated phacoemulsification procedure. Three months
later the UGH syndrome results. All the following except one may be found as a part of syndrome:

a. Anterior uvetitis

b. Hyphema.

c. Glaucoma.

d. Retinal detachment

Solution. d
The UGH syndrome is caused by mechanical excoriation of the angle or iris, or both, by the haptics or optic of an IOL and consists of
uveitis, glaucoma andhyphema (UGH). The inflammation is restricted to the anterior segment of the eye.
Ref –Kanski6thedi. Pg 339

Correct Answer. d

(73). Miotics such as pilocarpine, false is:

a. May worsen pupillary block by backward lens movement

b. causes pseudomyopia

c. Are not effective if the iris sphincler is ischemic

d. can cause iris cysts

Solution. a
Miotics may worsen papillary block by forward lens Movement.
Side effect:
• Uveitis
• Ciliary spasm
• myopia
• Cataract
• Iris cyst
• Retinal detachment
Ref – Kanski6thedi. Pg 422

Correct Answer. a

Copyright © 2014 Delhi Academy of Medical Sciences, All Rights Reserved. 29/76
(74). Synchysis of vitreous is seen in all except

a. Trauma

b. Age related

c. Posterior capsular rupture

d. Hypermetropia

Solution. d
Synchysis means vitreous liquefaction, seen in myopia, old age, trauma, retinal degeneration, posterior capsular rupture, posterior
capsulotomy.

Correct Answer. d

(75). All of the following will have false low reading with applanation tonometer except?

a. thin cornea

b. thick cornea

c. edematous cornea

d. keratoconus

Solution. b
Thick cornea overestimates IOP. Corneal edema also gives a falsely low pressure and patients with keratoconus often have thin corneas.
Ref - Glaucoma Medical Diagnosis and Therapy, 2ndedi, pg 98

Correct Answer. b

(76). Within which orbital bone are the optic foramen and canal contained?

a. frontal.

b. ethmoid.

c. lesser wing of sphenoid

d. greater wing of sphenoid

Solution. c
Blunt cranial trauma is often transmitted to the sphenoid, with traumatic optic neuropathy asa consequence.
Ref – Kanski6thedi. Pg 166

Correct Answer. c

(77). The study of choice for defining the presence or extent of intracranial involvement with optic nerve glioma is;

a. polytomography.

b. ultrasonography.

c. computed tomography (CT) scanning

d. magnetic resonance imaging (MRI

Solution. d
Magneticresonanceimaging (MRI) has been a major advancein the management of patients with optic nerve tumors.
Ref – Kanski6thedi. Pg 189

Correct Answer. d

Copyright © 2014 Delhi Academy of Medical Sciences, All Rights Reserved. 30/76
(78). Which of the following is / are primary determinants of intraocular pressure (IOP) except?

a. episcleral venous pressure.

b. rate of aqueous humor secretion

c. aqueous humor outflow facility.

d. relative pupillary block

Solution. d
As with any system of fluid dynamics, the equation pressure = flow * resistance defines the relationship between these variables. In the
eye, the driving pressure for aqueous outflow equals the difference between intraocular pressure (IOP) and episcleral venous pressure
(Ve = (aqueous flow / outflow facility), so, IOP = (aqueous flow / outflow facility) + Ve.
Aqueous flow, outflow facility, and episcleral venous pressure are thus the primary determinants of IOP Relative papillary block can
affect IOP by decreasing outflow facility, but is not a primary determinant of IOP.
Ref – Kanski6thedi. Pg 372

Correct Answer. d

(79). A pearly eyelid nodule with a central crater and associated inferior follicular conjunctivitis is the classic picture for;

a. phlyctenulosis

b. staphylococcal blepharoconjunctivitis

c. phthiriasis

d. molluscum contagiosum

Solution. d
Removal of the lesion is curative
Ref – Kanski6thedi. Pg 240

Correct Answer. d

Copyright © 2014 Delhi Academy of Medical Sciences, All Rights Reserved. 31/76
(80). What is false for Pseudotumor cerebri:

a. Is associated with permanent visual loss in 50 % of cases.

b. May be caused by drugs such as corticosteroids or tetracycline.

c. Has an indentifiable cause is almost all patients

d. Usually first appears with headache

Solution. c
Has an identifiable cause in almost all patients.
Also known as Benign Intracranial Hypertension.
Etiology:
1. Obesity
2. OCPs
3. Hypervitaminosis A
4. Tetracycline toxicity
5. Nalidixic acid
6. Amiodarone
Clinical features:
1. Headache
2. B/L papilloedema
3. U/L or B/L sixth nerve palsy leads to diplopia
4. Transient visual obscuration
5. Visual field defects: Enlargement of blind spot and later constriction of visual field.
Ref – Wills eye Manual, 6th edi. Pg 260

Correct Answer. c

(81). Which of the following organism typically causes the most severe complications of ophthalmia neonantorum?

a. Chlamydia trachomatis

b. Neisseria gonorrhea

c. Staphylococcus aureus

d. Streptococcus pneumonia

Solution. b
Neisseria gonorrhea can cause severe inflammation, hyperpurulence, endophthalmitis, and corneal perforation. It should be presumed in
an infant with hyperpurulence until proven otherwise.
Ref – Kanski6thedi. Pg 239

Correct Answer. b

(82). Which one of the following statements about fluorescein angiography (FA) and the blood – ocular barriers is true?

a. The most dreaded complication of FA is mortality from analphylactic shock

b. Fluorescein absorbs light in the yellow – green – range ---530 mm

c. After stimulation, fluorescein emits light in the blue range ---490 mm

d. The “ red – free “ filter is the initial filter through which white light passes before entering the eye

Solution. a
In fluoresceinangiography (FA), white light from the camera first passes through a blue filter. The blue light (wavelength of 490 nm) is
absorbed by thefluorescein molecules in the retinal and choroidal vasculature, stimulating them to emit yellow green light (530 nm). A
yellow – green filter is placed to block the blue light reflected from the eye, allowing the yellow – green light into the camera. Fluorescein
molecules (not bound to albumin), which have molecular weight< 600 daltons, can easily passthrough the spaces between endothelial
cells of the choriocapillaris but normally cannot leak through the tight junctions of the retinal pigment epithelium (RPE) or retinal
vascularendothelium (blood – retinal barrier). Anaphylactic shock is thought to occur at a rate of< 1 in 100,000 procedures.
Ref – Kanski6thedi. Pg 35

Correct Answer. a

Copyright © 2014 Delhi Academy of Medical Sciences, All Rights Reserved. 32/76
(83). Among men, each of the following constitutes a significant risk factor for the development of the condition in the figure except;

a. systemic hypertension

b. diabetes mellitus

c. elevated erythrocyte sedimentation rate

d. open - angle glaucoma

Solution. c
According to the eye disease Case – Control Study, elevated erythrocyte sedimentation rate is a risk for central retinal vein occlusion
(CRVO) only among women.
Ref – Kanski6thedi. Pg 584

Correct Answer. c

(84). Phenol red thread test is used for dry eye. This test:

a. Requires topical anesthetic agent

b. Volume of tears is measured as it changes color on contact with tears

c. If the color changes to blue it depicts surface mucin deficiency

d. Requires pH meter for reading the result

Solution. b
Volumeof tearsismeasuredasit changescoloroncontactwith tears
For Phenol red test, a 75-mm-long phenol-red impregnated thread with 3-mm bent end is placed in lower fornix for 15 seconds. When the
phenol red comes in contact with alkaline tears, it changes color from white to yellow-orange, yellow and then to red. The thread is
removed after 15 seconds and the red portion is measured from the very tip regardless of the fold. The results are interpreted as follows:
wet length <10 mm as severe dry eye, ≤19 mm as borderline dry eye and >20 mm as normal tear production
Ref – Kanski6thedi. Pg 208
Yanoffand Duker4/ep277

Correct Answer. b

Copyright © 2014 Delhi Academy of Medical Sciences, All Rights Reserved. 33/76
(85). The mechanism by which botulinum toxin works is:

a. sarcoplasmic capture of extracellular calcium.

b. direct blockage of post synaptic acetylcholine receptors.

c. inhibition of the formation actin – myosin complexes.

d. inhibition of release of acetylcoline from presynaptic nerve terminals.

Solution. d
Botulinum toxin acts by binding presynaptically to high-affinity recognition sites on the cholinergic nerve terminals and decreasing the
release of acetylcholine, causing a neuromuscular blocking effect.

Correct Answer. c

(86). Which one of the following constitutes a violotion of Hering’s law?

a. Which one of the following constitutes a violotion of Hering’s law?

b. Dissociated vertical deviation (DVD)

c. Brown’s syndrome

d. cyclic esotropia

Solution. b
In dissociated vertical deviation(DVD), the affected eye drifts up under cover. When the cover is shifted to the other eye, if Hering’s law
is followed, yoke innervations should generate a contralateral hypotropia. This is not seen in DVD for reasons that are unclear.
Ref – Kanski6thedi. Pg 766

Correct Answer. b

(87). The triad of adenoma sebaceum, mental retardation, and seizures is considered pathognomonic for;

a. neurofibromatosis.

b. Sturge – Weber syndrome

c. ataxia – telangiectasia

d. tuberous sclerosis

Solution. d
This is known as Vogt’s triade and is present in 30 % of patients with a tuberous sclerosis. Facial angiofibromas(adenoma sebaceum) are
present in at least 75 % of adults with tuberous sclerosis.
Ref – Wills eye Manual, 6th edi. Pg 7

Correct Answer. d

Copyright © 2014 Delhi Academy of Medical Sciences, All Rights Reserved. 34/76
(88). The predominant cell forms seen in the figure are

a. lymphocytes

b. epithelioid cells

c. polymorphonuclear leukocytes (PMNs)

d. macrophages

Solution. d
Note that“ granulomatous “ keratic precipitates (KPs) may ave a characteristic greasy, mutton –fat appearance, without actually being
granulomas themselves. Macrophages are responsible for the clinical appearance.
Ref – Wills eye Manual, 6th edi. Pg 358

Correct Answer. d

(89). Which of the following is not a common side effect of topical prostaglandin analogs?

a. eyelash hypertrichosis

b. conjunctival hyperemia

c. darkening of periocular skin

d. d iris atrophy / iris transillumination defects

Solution. d
Iris atrophy is not a common side effect of the prostaglandin analogs (e. g.,latanoprost /travoprost). All the other answers listed are
potential side effects of topical prostaglandin analogs.
Ref – Kanski6thedi. Pg 423

Correct Answer. d

(90). The nevus flammeus, which may be mistaken for a capillary hemangioma, is seen as a part of what systemic disorder?

a. neurofibromatosis.

b. Treacher Collins’ syndrome

c. von hippel’s disease.

d. Sturge – Weber syndrome

Solution. d
The nevus flammeus(port –wine stain) is a canvernoushemangiomaof the dermis. It can be seenin Sturge- webersyndrome,klippel –
Trenaunay – weber syndrome, and as anidiopathic finding. Somepathology textbooksalsoreferto the lessons as a dermaltelangiectasia.
Ref – Kanski6thedi. Pg 190

Correct Answer. d

Copyright © 2014 Delhi Academy of Medical Sciences, All Rights Reserved. 35/76
(91). Which collagen shields collagen 2 in vitreous from breakdown

a. 1

b. 3

c. 9

d. 10

Solution. c
Vitreous liquefaction, also known as syneresis, begins with the breakdown of collagen fibrils into smaller fragments. This liquefaction is
thought to occur because of a loss of "shielding" of type II collagen by type IX collagen. This process has no direct effect on the
development of vitreous hemorrhage unless it leads to the development of posterior
vitreous detachment (PVD). A PVD can protect against retinal neovascularization by
eliminating the scaffold for fibrovascular proliferation. Oxygen tension increases in the
posterior chamber in postvitrectomized eyes.

Correct Answer. c

(92). Which of the following is true for this condition:

a. Is initially associated with a clear lens or minimal opacity that may later become densely cataractous

b. is often bilateral

c. always requires lensectomy and vitrectomy

d. is associated with buphthalmos

Solution. a
PHPV is unilateral disorder associated with microphthalmia, and it has a variable severity of presentation. Some cases of PHPV may
simply be observed or treated with lensectomy alone.
Ref – Kanski6thedi. Pg 89

Correct Answer. a

Copyright © 2014 Delhi Academy of Medical Sciences, All Rights Reserved. 36/76
(93). During cataract surgery, intraoperatively the eye becomes stony hard following which intraocular contents start coming out of the eye.
What are the predisposing factors for such a scenario?

a. younger age

b. hyperopia

c. elevated intraocular pressure during surgery

d. hypertension

Solution. d
Hypertension is the only risk factor for suprachoroidal detachment and hemorrhage; additional risk
factors include advanced age, glaucoma, myopia, aphakia, cardiovascular disease, SturgeWeber-
associated choroidal hemangioma, andtachycardia. Transient hypotony during surgery may cause rupture of a posterior ciliary or
choroidal artery.

Correct Answer. d

(94). What is false for Optic Nerve hypoplasia:

a. is associated with diabetes in child

b. Often appears with a double -ring sign

c. Is associated with trans-sphenoidal encephalocoele

d. May be associated with panhypopituitarism.

Solution. a
Optic nerve hypoplasia is characterized by decreased number of optic nerve axons. It may present as an isolated anomaly or be
associated with midline brain structural defect, such as septum pellucidum absence, agenesis of corpus callosum, cerebral hemisphere
abnormalities, or pituitary gland abnormalities. Septo – Optic dysplasia (de Morsier syndrome) is used to describe the association
between optic nerve hypoplasia and the absence of septum pellucidum and agenesis of corpus callosum. Many prenatal risk factors have
been reported: young maternal age, diabetes mellitus, maternal use of recreational drugs, anticonvulsants, or antidepressants; viral
infections during pregnancy; fetal alcohol syndrome.
Ref – Wills eye Manual, 6th edi. Pg 245

Correct Answer. a

(95). Which of the following slit lamp biomicro scopy technique uses property of total internal reflection

a. retroillumination

b. scleral scatter

c. specular reflection

d. direct illumination

Solution. b
Specular reflection shows abnormality of endothelium such as guttata and reduced cell density
Ref – Kanski6thedi. Pg 2

Correct Answer. b

Copyright © 2014 Delhi Academy of Medical Sciences, All Rights Reserved. 37/76
(96). What is the magnification of indirect ophthalmoscope done with 20D lens for a patient with myopia -5D

a. 3

b. 3. 25

c. 3. 5

d. 4

Solution. b
In myopia we add as power is more in myopia its (60+5)/20 for myopia

Correct Answer. b

(97). Which is false for the hyaloid artery:

a. Is a branch of the ventral ophthalmic artery.

b. Anastomosis with the tunica vasculosa lentis.

c. Begins to regress in the third trimester.

d. May persist on the disk as a PHPV and Bergmister papilla are congenital remnant of the hyaloid arterial system.

Solution. a
Bergmeisterpapilla : it is seennear the optic disc.
Mittendorf dots: at posterior part of lens
The canal joining the two is called Cloquet canal
Bergmeister’s papilla.
The hyaloid artery is a branch of the dorsal ophthalmicartery.

Correct Answer. a

(98). A 41 – year – old patient has a history of bilateral recurrent inflammation of the superior limbus and conjunctiva. The probable diagnosis
is

a. Thygeson’s superficial punctuate Keratopathy

b. Moren’s ulcer

c. Superior limbic keratoconjuctivitis

d. Terrien’s marginal degeneration

Solution. c
Superior limbic keratoconjunctivitis(SLK) of Theodore is a bilateral, recurrent inflammation of the superior limbus, tarsal and bulbar
conjunctiva.It is seen more commonlyin women 30-55 years of age. SLK is associated with thyroid and collagen vascular diseases.
Silvernitrate (0.5 %) solution has been used in past. Resection or recession of the abnormal conjunctiva is very successful. Cryotheraphy
and thermopathy have also been used. Bandage contact lens,and pressure patching may be helpful in temporarily relieving symptoms.
Ref – Kanski6thedi. Pg 239

Correct Answer. c

Copyright © 2014 Delhi Academy of Medical Sciences, All Rights Reserved. 38/76
(99). The strongest attachment of vitreous is

a. Weigert ligament

b. Fovea

c. Optic disc

d. Ora serrata

Solution. d
All are strong attachments of vitreous but oraverrata is the strongest.
Ref – Kanski6thedi. Pg 89

Correct Answer. d

(100). Roth spots are seen in

a. Leukemia

b. Hodgkin Lymphoma

c. Non Hodgkin lymphoma

d. Anemia

Solution. a
Roth Spots are defined as a white centered retinal hemorrhage and are associated with multiple systemic illnesses, most commonly
bacterial endocarditis.
Theymay be observed in leukemia, diabetes, pernicious anemia, ischemic events, hypertensive retinopathy and rarely in HIV retinopathy.
Ref – Wills eye Manual, 6th edi. Pg 298

Correct Answer. a

(101). In which condition is the underlying pathology stained with alcian blue?

a. Schnyder dystrophy

b. granular dystrophy

c. lattice dystrophy

d. macular dystrophy

Solution. d
Macular dystrophy-Mucopolysaccharide deposit- Alcian blue
Granular dystrophy-Hyaline deposit -Mason trichrome
Lattice dystrophy-Amyloid deposit -Congo red
Ref – Kanski6thedi. Pg 291

Correct Answer. d

Copyright © 2014 Delhi Academy of Medical Sciences, All Rights Reserved. 39/76
(102). Which is true for the following Lens condition

a. Is usually associated with previous lens trauma

b. Is typically located superiorly

c. Is typically associated with normal zonular attachments

d. Is often associated with cortical lens opacifications

Solution. d
A lens coloboma is a wedged –shaped defect or indentation of the lens periphery that occurs as an isolated anomaly or secondary to the
lack of ciliary body or zonular development. Lens colobomas are typically located inferiorly and may be associated with colobomas of the
uvea. Cortical lens opacification or thickening of the lens capsule may appear adjacent to the defect.
Ref – Kanski6thedi. Pg 70

Correct Answer. d

(103). A patient presents with poor vision in one eye. Macular edema and a peripheral retinal vascular lesion that contains feeder vessels are
found in the periphery. Test show an abnormality in the 3p25-26 gene. Which of the following would you not except to find?

a. Retinal capillary hemangioma

b. Central nervous system hemangio blastoma.

c. Cardiac rhabdomyoma

d. Renal cell carcinoma

Solution. c
Rhabdomyomais not associated with vonHippel – Lindau.
Von Hippel-Lindau Syndrome
1. Hereditarypattern- AD
2. Skin features (not prominent)
• Café au lait
• Melanocytic nevi
3. CNSfeatures
• Hemangioblastoma
4. Ocularfeatures
• Capillary hemangioma of retina
• Hypertensive retinopathy (pheochromocytoma) (CNS hemangioblastoma)
5. Others
• Visceral tumors:
• Cysts of kidney, pancreas, liver, epididymis,ovary and lungs
• Hypernephroma
• Pheochromocytoma
Ref – Textbook of Clinical Ophthalmology 3th edi. Pg 329

Correct Answer. c

Copyright © 2014 Delhi Academy of Medical Sciences, All Rights Reserved. 40/76
(104). Treatment of post cataract surgery endophthalmitis in a patient with visual acuity less than hand movements is

a. intravitreal antibiotic

b. pars plana vitrectomy

c. topical antibiotics

d. topical steroids

Solution. b
According to endophthalmitis vitrectomy study, if presenting vision is less than hand movements, then pars plana vitrectomy else
intravitralanbibiotics are given There is no role of systemic antibiotics.
Ref – Wills eye Manual, 6th edi. Pg 111

Correct Answer. b

(105). A diabetic woman who continued alcohol consumption during gestation gave birth to a child with poor vision, absence of septum
pellucidum, hypopituitarism, agenesis of corpus callosum. Which feature is most likely to be present in eyes

a. Persistent hyperplastic primary vitreous

b. Optic nerve hypoplasia

c. Optic disc drusen

d. Bergmister papilla

Solution. b
Optic nerve hypoplasia is characterized by decreased number of optic nerve axons. It may present as an isolated anomaly or be
associated with midline brain structural defect, such as septum pellucidum absence, agenesis of corpus callosum, cerebral hemisphere
abnormalities, or pituitary gland abnormalities. Septo – Optic dysplasia (de Morsier syndrome) is used to describe the association
between optic nerve hypoplasia and the absence of septum pellucidum and agenesis of corpus callosum.
Many prenatal risk factors have been reported: young maternal age, diabetes mellitus, maternal use of recreational drugs,
anticonvulsants, or antidepressants; viral infections during pregnancy; fetal alcohol syndrome.
Optic nerve head drusen are globular, often calcified, hyaline bodies located within the optic nerve head
Ref – Kanski6thedi. Pg 83

Correct Answer. b

(106). All are causes of amaurosis fugax except

a. CRAO

b. Migraine

c. Giant cell arteritis

d. SDiabetic retinopathy

Solution. d
Amaurosisfugax is a painless temporary loss of vision in one or both eyes Ref – Kanski6thedi. Pg 591

Correct Answer. d

Copyright © 2014 Delhi Academy of Medical Sciences, All Rights Reserved. 41/76
(107). A mother brings in her 11-months- old child because of a “ funny” reflex in one eye, described as a cat’s- eye reflex. Retinoblastoma is
suspected. Which of the following is the most common presentation of retinoblastoma?

a. Pseudohypopion.

b. Neovascular glaucoma

c. A red eye

d. Leucokoria (white pupil).

Solution. d
Leukokoria and strabismus are the most common presenting signs of retinoblastoma. Mode of presentations are
1. Leukocoria (60%): It is the most common mode of presentation.
2. Strabismus (20%): It is the second most common mode of presentation.
3. Secondary glaucoma
4. Pseudouveitis.
5. Orbital inflammation - mimicking orbital cellulitis.
6. Proptosis.
7. Heterochromia iridis, rubeosisiridis
Ref – Wills eye Manual, 6thedi. Pg 170

Correct Answer. d

(108). Which one of the following does not occur with high myopia

a. Subnormal visual acuity

b. Suboptimal binocularity

c. Abnormal color vision

d. Image magnification

Solution. d
Image minificationis one of the functional defects ofhigh myopia.
Ref – Kanski6thedi. Pg 654

Correct Answer. d

(109). The inner retinal circulation’s deepest level of penetration is the;

a. ganglion cell layer

b. inner plexiform layer

c. inner nuclear layer

d. rods and cones

Solution. c
Retinal blood vessels usually do not extend beyond the inner third of the inner nuclear layer
Ref – Wills eye Manual, 6thedi. Pg 293

Correct Answer. c

Copyright © 2014 Delhi Academy of Medical Sciences, All Rights Reserved. 42/76
(110). Systemic disorders associated with angioid streaks include all except?

a. Paget’s disease of bone

b. Pseudoxanthoma elasticum

c. Ethler’s –Danlos syndrome

d. high myopia

Solution. d
Sickle cell (SC) anemia is another potential systemic disease associated with angioid streaks. Up to 50 % of patients with angioid streaks
have no identifiable systemic illness. High myopia is not associated with angioid streaks but may feature lacquer cracks, which are
similar histopathologically.
Ref – Wills eye Manual, 6th edi. Pg 296

Correct Answer. d

(111). A one-month old baby is brought with complaints of photophobia and watering. Clinical examination shows normal tear passages and
clear but large cornea as in the figure. The most likely diagnosis is:

a. Congenital dacryocystitis

b. Interstitial keratitis

c. Keratoconus

d. Buphthalmos

Solution. d
Congenital glaucoma
CLINICAL FEATURES:
General- Photophobia, blepharospasm, lacrimation and eye-rubbing
Corneal signs- Corneal edema
Corneal enlargement - (>13mm)
Haab'sStriae - break in DM
Buphthalmos- Blue and thin sclera
Iridodonesis
Deep AC
Flat lens; which may sub-luxate
Glaucoma cupping- Enlarged C:D ratio may be either due to neuronal loss or enlargement of scleral canal
IOP- Raised but neither marked nor acute
Ref – Wills eye Manual, 6th edi. Pg 207

Correct Answer. d

Copyright © 2014 Delhi Academy of Medical Sciences, All Rights Reserved. 43/76
(112). Irreversible structural damage occurs in retina after what duration of total ischemia?

a. 15 to 20 minutes

b. 30 to 40 minutes

c. 45 to 60 minutes

d. 90 to 100 minutes

Solution. d
Studies in monkeys whose central retinal arteries were ligated established that irreversible damage to retina occurs after 97 minutes of
total ischemia.
Ref – Kanski6thedi. Pg 591

Correct Answer. d

(113). The site of origin of neural impulses leading to a rightward saccade is the;

a. right frontal lobe

b. left frontal lobe

c. right parietooccipital lobe

d. left parietooccipital lobe

Solution. b
Left frontal eye field in frontal lobe controls right PPRF, which is responsible for right horizontal movements
Ref – Wills eye Manual, 6th edi. Pg 250

Correct Answer. b

(114). The dorsal midbrain syndrome is associated with all of the following except;

a. upward gaze paresis

b. light reflex abnormalities

c. light – near dissociation

d. paradoxic optokinetic nystagmus (OKN

Solution. d
Upward gaze paresis, light – near dissociation, lid retraction and convergence – retractionnystagmus are all features of the dorsal
midbrain (Parinaud’s) syndrome. Furthermore, skew deviation and papilledema may be seen, depending on the etiology. Lid retraction –
the Collier sign – may worsen with attempted upgaze. Convergence – retraction nystagmus is also a response to an effort at upgaze that
triggers medial rectus contractions. Hence, this form of “nystagmus “ is worsened by upward optokinetic nystagmus (OKN) testing.
Paradoxic OKN is not a feature.
Ref – Wills eye Manual, 6th edi. Pg 251

Correct Answer. d

Copyright © 2014 Delhi Academy of Medical Sciences, All Rights Reserved. 44/76
(115). A 60-year-old man is unable to open his eye because of a rare neuromuscular disease. Which of the following muscles would most likely
be paralyzed?

a. Orbicularis oculi

b. Orbicularis oris

c. Frontalis

d. Levator palpebrae superioris

Solution. d
Levatorpalpebraesuperioris
• The levatorpalpebraesuperioris muscle opens the eye by elevating the upper eyelid.
• The orbicularis oculi closes the eye, the orbicularis oris closes the lips, the frontalis elevates the eyebrow, and the superior rectus
elevates the eyeball.
Ref – Kanski6thedi. Pg 133

Correct Answer. d

(116). Identify the device used in measuring visual acuity in children

a. Tellers acuity test

b. opto-kineto-nystagmogram

c. Cardiff test

d. Visual evoked response

Solution. b
Optokinetonystagmogram (OKN) is thought to occur because the eyes are trying to keep a moving image stationary on the fovea.The
reflex is fully developed by 5-6 months, but is present in a crude form in newborn infants.When a subject views a rotating striped drum,
his or her eyes involuntarily follow a stripe with a “slow eye movement” (pursuit), then return with a fast eye movement (saccade) to
fixate on a new stripe.The whole cycle can be repeated indefinitely as long as the subject views the moving drum.
Ref – Kanski6thedi. Pg 736

Correct Answer. b

Copyright © 2014 Delhi Academy of Medical Sciences, All Rights Reserved. 45/76
(117). Potential complications of carotid – cavernous fistulae include all except;

a. retinal neovascularization

b. cataract.

c. enophthalmous

d. corneal ulceration

Solution. c
Iris and posterior segment neovascularization, as well as rapidly progressive cataract. may all be seen as complications of the ischemic
oculopathy that these fistulae generate. Corneal exposure caused by proptosis is another potential complication of carotid – cavernous
fistula
Ref – Wills eye Manual, 6th edi. Pg 160

Correct Answer. c

(118). Pupillary dilation may not be a sign of oculomotor nerve palsy in which of the following disorders?

a. uncal herniation.

b. diabetic microvascular disease

c. basilar meningitis

d. cerebral aneurysm

Solution. b
Oculomotor nerve palsy usually includes ptosis, Limitation of ocular motility, and papillary abnormalities. In uncal herniation and basilar
meningitis, pupil dilation may be the only sign of nerve palsy. In the case of cerebral aneurysm, the pupil is usually involved, along with
other functions of the third nerve. If the pupil is not involved, it is less likely that an aneurysm is the cause of palsy. Total oculomotor
palsywith a sparedpupil in elderly patients usually suggests a microvascular etiology.
Ref – Kanski6thedi. Pg817

Correct Answer. b

(119). A patient presents 6 hours after receiving a prescription for topical antibiotic for a suspected bacterial bleparo conjunctivitis. The patient
complains of itching and tearing, and an examination documents severe chemosis and mild hyperemia. The most likely diagnosis is;

a. antibiotic resistance

b. toxic follicular conjunctivitis

c. toxic papillary conjunctivitis

d. anaphylactoid reaction

Solution. d
An anaphylactoid reaction is rapid in onset with itching, conjunctival erythema, and chemosis and is usually caused by penicillin,
bacitracin, sulfacetamide, or anesthetics.
Ref – Wills eye Manual, 6thedi. Pg 111

Correct Answer. d

Copyright © 2014 Delhi Academy of Medical Sciences, All Rights Reserved. 46/76
(120). Which of the following regarding cavernous hemangioma of the orbit is false?

a. Women are more affected than men

b. It is the most common etiology of neoplastic unilateral proptosis in adults.

c. A – scan ultrasonography reveals high internal reflectivity.

d. There is generally an associated ocular bruit.

Solution. d
These lesions generally do nothave the high arterial blood flow usually associatedwith bruits. Their composition – denselypacked
vascularchannels – leads to a high internal reflectivityon ultrasonography. Ocularbruits can be associated withcarotid –
canvernousfistulas and sphenoid wing dysplasia
Ref – Wills eye Manual, 6th edi. Pg 160

Correct Answer. d

(121). The mean value for outflow facility in normal eyes is;

a. 0. 05 uL /min /mm Hg

b. 0. 15 uL /min /mm Hg.

c. 0. 28 uL /min /mm Hg.

d. 0. 48 uL /min /mm Hg

Solution. c
Outflow facility in normal eyes ranges from 0.22 to 0.28 uL /min /mm Hg, and decreases with age, ocular surgery, and trauma. Patients
with glaucoma often have decreased outflow facility.
Ref – Wills eye Manual, 6th edi. Pg 204

Correct Answer. c

(122). The most common clinical finding in infants with congenital rubella syndrome is;

a. pigmentary retinopathy

b. patent ductus arteriosus

c. sensorineural hearing loss

d. mental retardation

Solution. c
Many cases of “ oculoauditorysyondrome “ are caused by rubella

Correct Answer. c

(123). A 65 –year –old woman is examined as part of a routine annual checkup. On retroillumination of the cornea, flecklike deposits in the deep
corneal stroma are detectable centrally. Visual acuity is normal, and there are no other ocular findings. Themost likely diagnosis is;

a. Hassall – Henle bodies

b. Cornea guttae

c. Cornea farinata

d. Arcus senilis

Solution. c
Hassal Henle bodies and guttate are seen in endothelium
Arcus senilis is seen in anterior stroma/bowmans membrane
Ref – Wills eye Manual, 6th edi. Pg 110

Correct Answer. c

Copyright © 2014 Delhi Academy of Medical Sciences, All Rights Reserved. 47/76
(124). A patient complains of maceration of skin of the lids and conjunctiva redness at the inner and outer canthi as in the figure. Conjunctival
swab is expected to show:

a. Slaphylococcus aureus

b. Streptococcus viridans

c. Streptococcus pneumonae

d. Moraxella- Axenfeld diplobacilli

Solution. d
Chronic conjunctivitis characterized by mild grade inflammation confined to conjunctiva and lid margins near the angles; associated with
maceration of surrounding tissue.
Etiology:
1. Moraxella axenfeld (diplobacillary conjunctivitis)
2. Staphylococcus aureus.
Ref – Wills eye Manual, 6th edi. Pg 110

Correct Answer. d

Copyright © 2014 Delhi Academy of Medical Sciences, All Rights Reserved. 48/76
(125). All are the following are true for the syndrome associated with this condition except

a. defect in Type IV basement membrane collagen

b. renal failure

c. AR

d. megalocornea

Solution. c
Alport’s syndrome is characterized by:
X-linked dominant inheritance
defect in Type IV basement membrane collagen
renal failure secondary to glomerular dysfunction
Ocular Features
megalocornea
posterior polymorphous dystrophy
anterior lenticonus
anterior sub-capsular cataract
corneal arcus
peripheral retinal flecks
Ref – Kanski6thedi. Pg 70

Correct Answer. c

(126). Which one of the following ganglia serves as home for cell bodies providing postganglionic efferent innervation to the lacrimal gland?

a. geniculate.

b. superior cervical

c. ciliary.

d. pterygoopaltine.

Solution. d
The pterygopalatineganglionreceives parasympathetic fibers from the greatersuperficialpetrosalnerve a division of CNVII)
Ref – Kanski6thedi. Pg151

Correct Answer. d

Copyright © 2014 Delhi Academy of Medical Sciences, All Rights Reserved. 49/76
(127). Which of the following statements about monocular diplopia is false?

a. monocular diplopia is usually relieved by covering either eye.

b. It can be caused by keratoconus.

c. It can be caused by lenticonus.

d. It can be caused by high astigmatism

Solution. a
Binocular diplopia is relieved by covering one eye. Monocular diplopia is relieved by covering the affected eye. All of the other answers
are potential causes for monocular diplopia.
Ref – Kanski6thedi. Pg 735

Correct Answer. a

(128). The organism responsible for approximately 25 % of posttraumatic endophthalmitis is;

a. Staphylococcus aureus

b. Streptococcus pneumonia

c. Aspergillus.

d. Bacillus cereus.

Solution. d
Postraumaticendophthalmitis has uniquely high percentage of Bacillus species. especially B. cereus, represented etiologically. Estimated
have ranged from 20 % to 25%, and the organisms seem to be particulary associated with retained metallic foreign body, as well as farm
orsoil – related injuries. B. cereus endophthalmitis can be extremely fulminant. The incidence of Staphylococcus epidermidis
endophthalmitis is slightly greater (about 30 % of posttraumatic endophthalmitis).
Ref – Kanski6thedi. Pg 483

Correct Answer. d

(129). All of the following are conditions that may be associated with nongranulomatous ocular inflammation except;

a. Choroidal melanoma

b. Juvenile rheumatoid arthritis.

c. Trauma.

d. Cryptococcal endophthalmitis

Solution. d
juvenile rheumatoid arthritis (JRA) and Reiter’s syndrome are specially characterized by recurrent or chronic nongranulomatous uveitis.
whereas choroidalmelanoma and ocular trauma may incite secondary ocular inflammation, which is nearly always nongranulomatous. (In
the case of a retained intraocular foreign body after trauma, there may be a granulomatous component).Cryptococcalendophthalmitis,
like other fungal causes of ocular inflammation, features marked granulomatous inflammation.
Ref – Kanski6thedi. Pg 443

Correct Answer. d

Copyright © 2014 Delhi Academy of Medical Sciences, All Rights Reserved. 50/76
(130). Afferent pupillary fibers from the optic tract exit at;

a. the lateral geniculate body (LGB)

b. the pretectal olivary nuclei.

c. the occipital cortex.

d. the frontal lobe.

Solution. b
Pupillary fibers from the optic tract exit before reaching the lateral geniculate body (LGB) and exit into the pretectalolivary nuclei.
Ref – Kanski6thedi. Pg 812

Correct Answer. b

(131). Which one of the following is not considered a risk factor for the development of bacterial keratitis

a. drug abuse

b. keratoconus

c. diabetes mellitus

d. dry eye syndrome

Solution. b
Risk factors for bacterial keratitis are those that cause disruption of the integrity of the corneal epithelium.
The most common risk factor for bacterial keratitis is contact lens wear Other predisposing factors include: trauma, contaminated ocular
solutions, changes in the corneal surface (from dry eye, eyelid misdirection, and exposure), altered ocular defense mechanisms (from
topical and systemic immune suppression), loose sutures with adjacent infections (blepharitis and viral keratitis), and corneal edema.
Ref – Kanski6thedi. Pg 254

Correct Answer. b

(132). The most common cause of third nerve palsy in the pediatric population is:

a. congenital.

b. traumatic

c. inflammatory

d. migraine

Solution. a
The etiologies for pediatric third nerve palsy, in descending frequency, are congenital, traumatic, inflammatory, migranious, and
neoplastic.
Ref – Kanski6thedi. Pg 816

Correct Answer. a

Copyright © 2014 Delhi Academy of Medical Sciences, All Rights Reserved. 51/76
(133). Identify the test (Image question)

a. FM 100 Hue test

b. Edridge Green lantern

c. Ishihara pseudoisochromatic plates

d. Anomaloscope

Solution. c
Color Vision Tests
1. Quantitative
• Farnsworth-Munsell 100 hue test: •
• Nagel’s anomaloscope:
2. Qualitative
Pseudoisochromatic color plate test: Examples: Ishihara, AO Hardy Rand Rittler
Ref – Kanski6thedi. Pg 15

Correct Answer. c

(134). Which of the following is / are universal findings in patients with pseudotumor cerebri except?

a. b/l disc edema

b. increased intracranial pressure

c. normal neurologic examination

d. normal neuroimaging studies

Solution. c
Pseudotumorcerebriischaracterized by (i) increased intracranial pressure on lumbar puncture,(ii) normal neuroimaging studies (although
the ventricles may be small), and (iii) normal cerebrospinal fluid (CSF). Although the neurologic examination is usually normal, sixth
nerve palsy may occur with increased intracranial pressure of any etiology.
Ref – Kanski6thedi. Pg 790

Correct Answer. c

Copyright © 2014 Delhi Academy of Medical Sciences, All Rights Reserved. 52/76
(135). The most common site for primary nonocular tumors to develop metastasis in or around the eye in adults is the;

a. orbit.

b. choroid.

c. retina.

d. optic nerve.

Solution. b
In adults, the choroids are the most common site of ocular / periocular metastasis.In children, the orbit is more commonly involved.
Ref – Kanski6thedi. Pg 526

Correct Answer. b

(136). Abnormally heavy trabecular meshwork (TM) pigmentation is associated with all except;

a. pseudoexfoliation

b. pigment dispersion

c. previous trauma or surgery

d. anterior segment dysgenesis

Solution. d
Heavy trabecular meshwork (TM) pigmentation is not a feature of anterior segment dysgenesis. It should suggest the differential
diagnosis of pseudoexfoliation, pigment dispersion, previous inflammation or surgery, or uveal melanoma.
Ref – Textbook of Clinical Ophthalmology 3th edi. Pg 81

Correct Answer. d

(137). The ptosis associated with Marcus Gunn’s syndrome is because of aberrant connections between the levator muscle and which cranial
nerve;

a. V

b. VII

c. IX

d. X

Solution. a
Levtorinnervations in Marcus Gunn’s syndrome is derivedfromthe trigeminal supply (CNV) to the pterygoidsandmasseters.
Ref – Textbook of Clinical Ophthalmology 3th edi. Pg 349

Correct Answer. a

(138). Which one of the following describes the stereotypic patient with pseudotumor cerebri?

a. 75- year-0ld woman with history of TIAs

b. 58-year-old man with a type A personality

c. 35-year-old overweight woman

d. 15-year-old black man with a poor diet

Solution. c
Patients with pseudotumorcerebri have a distinctive profile. They are typically obese women between the ages of 20 and 40 years. The
exact etiology of this condition is unknown. A hormonal imbalance has been hypothesised because pseudotumorcerebri may be
exacerbated with pregnancy.
Ref – Wills eye Manual, 6th edi. Pg 394

Correct Answer. c

Copyright © 2014 Delhi Academy of Medical Sciences, All Rights Reserved. 53/76
(139). Which is not a free radical scavenger in the lens

a. glutathione

b. vitamin C

c. vitamin A

d. catalase

Solution. c
In addition to glutathione, vitamin C and E, catalase, superoxide dismutase are protective against oxidative damage to the lens from free
radicals.
Ref – Textbook of Clinical Ophthalmology 3th edi. Pg 2

Correct Answer. c

(140). A patient has iop of 24 mm Hg with thick cornea and normal visual fields and optic disc .What should be the management of such patient?

a. Observation

b. Topical antiglaucoma meds

c. Trabeculectomy

d. Laser trabeculopasty

Solution. a
PRIMARY OPEN ANGLE GLAUCOMA [POAG]
Spectrum

Risk factors
• Heredity
• Age - common elderly
• Myopes - more common
• Diabetics- more common
• Hypertension- more common
• Cigarette smoking
• Thyrotoxicosis
Thick cornea give false high IOP reading. In absence of any risk factors in tis ocular hypertensive patient, he should be observed.
Ref – Textbook of Clinical Ophthalmology 3th edi. Pg 61

Correct Answer. a

(141). Which is incorrect regarding Fasanella- Servat operation

a. is used on correction of ptosis in which levator function is 5 –12 mm

b. is restricted to cases with ptosis of 1 – 2 mm

c. includes excision of part of Muller’s muscle

d. is commonly complicated by corneal abrasion

Solution. a
This operation is suitable if levator function exceeds 10 mm, for example in Horner’s syndrome. For levator function between 5 and 10
mm, levator resection is appropriate.
Ref – Wills eye Manual, 6thedi. Pg 135

Correct Answer. a

Copyright © 2014 Delhi Academy of Medical Sciences, All Rights Reserved. 54/76
(142). Which of the following terminal branches of the ophthalmic artery anastomose with branches of the external carotid system?

a. the supraorbital artery.

b. the lacrimal artery

c. the dorsonasal artery

d. the angular artery.

Solution. d
These anastomoses may prove critical in the sittings of high grade carotid stenosis. (The angular artery is the terminalbranch of the
facial artery, a branch of the external carotid system).
Ref – Kanski6thedi. Pg 566

Correct Answer. d

(143). Identify the instrument

a. Goldmann tonometer

b. Perkin tonometer

c. Schiotz tonometer

d. Rebound tonometer

Solution. a
Goldmann tonometry is considered to be the gold standard IOP test and is the most widely accepted method. A special disinfected prism
is mounted on the tonometer head and then placed against the cornea. The examiner then uses a cobalt blue filter to view two green
semi circles. The force applied to the tonometer head is then adjusted using a dial connected to a variable tension spring until the inner
edges of the semicircles meet.
Ref – Textbook of Clinical Ophthalmology 3th edi. Pg 51

Correct Answer. a

Copyright © 2014 Delhi Academy of Medical Sciences, All Rights Reserved. 55/76
(144). The most common fundus finding in a patient with acute leukemic oculopathy is;

a. choroidal infiltration

b. nerve fiber layer hemorrhages

c. cotton – wool spots

d. Roth’s spots

Solution. b
Flame – shaped nerve fibrehemorrhages are the one most common funduscopic finding. White – centered hemorrhages
(Roth’s spots), cotton –wool spots, optic disc swelling, and perivascular infiltration also may be seen but are less common.
Chorodial infiltration is usually not ophthalmoscopically apparent
Ref – Textbook of Clinical Ophthalmology 3th edi. Pg 431

Correct Answer. b

(145). Systemic disease associations for scleritis include all except;

a. rheumatoid arthritis

b. Wegener’s granulomatosis

c. Polyarteritis nodosa

d. Inflammatory bowel disease

Solution. d
Scleritis is frequently associated with autoimmune connective tissue diseases such as systemic lupus erythematosus (SLE), rheumatoid
arthritis,polyarteritisnodosa, or Wegener’s granulomatosis. Inflammatory bowel disease also has been reported in conjunction with
peripheral ulcerative keratitis.
Ref – Kanski6thedi. Pg 325

Correct Answer. d

(146). Which would be the best single medication for treatment of primary open-angle glaucoma (POAG) in a 69- year- old patient with aphakia
with chronic bronchitis, labile hypertension, and chronic depression treated with monoamine oxidase inhibitors (MAOLs)?

a. epinephrine.

b. dipivefrin.

c. pilocarpine.

d. timolol.

Solution. c
Because epinephrine has been associated with a reversible cystoids maculopathy (in approximately 25% of patients with aphakia who
have been chronically treated), both epinephrine and the prodrug Dipivalyl epinephrine (dipivefrin) would be relatively contraindicated
in this case. Furthermore, alpha-agonists (epinephrine is an alpha- and beta- agonist) are contraindicated in patients who may have an
abnormally increased sensitivity to their cardiovascular effects caused by the use of drugs such aas monoamine oxidase inhibitors
(MAOIs), tricyclic antidepressants, cocaine, or reserpine. In general, depression may be a side effect of beta-blockers and
arelativecontraindications to acetazolamide therapy. Therefore, the best single alternative drug in this case would be pilocarpine.
Ref –Kanski6thedi. Pg 421

Correct Answer. c

Copyright © 2014 Delhi Academy of Medical Sciences, All Rights Reserved. 56/76
(147). What is the most common ocular presentation of shaken baby syndrome?

a. retinal hemorrages

b. cotton wool spots

c. retinal detachment

d. retinal folds

Solution. a
Shaken baby syndrome is a form of nonaccidental trauma, typically in a baby less than a year old, frequently less than 6 months of age.
Systemic signs include bradycardia, apnea, hypothermia, lethargy, seizures, and bulging fontanelles. Skin bruises may be present,
particularly on the upper arms, chest, or thighs, as may long-bone fractures and subdural and subarachnoid hemorrhages. Ocular signs
include retinal hemorrhages, cotton-wool spots, retinal folds, and hemorrhagic schisis cavities.
Ref – Kanski6thedi. Pg 622

Correct Answer. a

(148). Drainage implants is less useful in patients with:

a. Glaucoma caused by intraocular tumors

b. Extensive anterior conjuctival scarring

c. Extensive filtration angle scarring

d. Aphakic glaucoma.

Solution. a
A drainage implant is not useful for glaucoma caused by the intraocular tumors.
Artificial filtering Shunts:
It is used in cases where conventional filtering procedure has failed or likely to fail. These are plastic devices which create a
communication between the anterior chamber and sub- tenon’s space.
a) Krupin- Denver Valve
b) Molteno Implant
c) Schoket Implant
d) Ahmed Glaucoma Valve (AGV)
Ref – Textbook of Clinical Ophthalmology 3th edi. Pg 99

Correct Answer. a

(149). Combined central retinal artery and vein occlusion is suggested by all the following findings except:

a. Severe visual loss (bare to no light perception).

b. Few or no cherry – red spots

c. Features of central retinal artery occlusion and retinal hemorrhages is all four quadrants

d. Usually an associated systemic or local disease

Solution. b
Combined artery and veinobstruction show a cherry – red spot combinedwith featuresof a central retinalveinobstruction.
Associatedsystemic or localdisease is the rule. Collagen vasculardisorders, leukemia, orbital trauma, retrobulbar injections,
andmucormycosishave been implicated.
Ref – Kanski6thedi. Pg 590

Correct Answer. b

Copyright © 2014 Delhi Academy of Medical Sciences, All Rights Reserved. 57/76
(150). In which bone does the lacrimal gland fossa lie?

a. frontal

b. lacrimal

c. maxillary

d. zygomatic

Solution. a
Thelacrimal gland fossalies in theanterolateral orbital roof within the zygomatic process of thefrontal bone.
Ref – Textbook of Clinical Ophthalmology 3th edi. Pg 347

Correct Answer. a

(151). Itentify the condition

a. Trichiasis

b. Distichiasis

c. Proptosis

d. Ptosis

Solution. b
Trichiasis – misdirected eyeashes
Proptosis – protrusion of eyeball beyond orbital margins
Ptosis – drooping of eyelid
Ref – Textbook of Clinical Ophthalmology 3th edi. Pg 347

Correct Answer. b

(152). Which of the following features is not consistent with a parietal lobe lesion?

a. agnosia.

b. right – left confusion

c. optokinetic nystagmus (OKN) abnormalities.

d. homonymous hemianopia denser superiorly

Solution. d
Parietal lobe lesions are associated with agnosia and right left confusion.
Slow pursuit movement of same side are also affected. It causes homonymous hemianopia denser inferiorly.
Ref – Wills eye Manual, 6thedi. Pg 394

Correct Answer. d

Copyright © 2014 Delhi Academy of Medical Sciences, All Rights Reserved. 58/76
(153). Which viral infection can cause an increased risk of corneal xerophthalmia in a malnourished child?

a. Herpes virus

b. Measles virus

c. Smallpox

d. Human immunodeficiency virus

Solution. b
The catabolic state associated with measles infection can lead to severe corneal xerophthalmia.
All ocular manifestations of vitamin-A deficiency including not only the structural changes affecting the conjunctiva, cornea and retina
but also biophysical disorders of retinal rods and cone function.
Etiology: Dietary deficiency of vitamin A or its defective absorption.

XS Corneal scar due to xerophthalmia


XF Xerophthalmic Fundus: Focal loss of RPE cells lead to Uyemura spots.
Ref – Textbook of Clinical Ophthalmology 3thedi. Pg 493

Correct Answer. b

(154). The indications for treatment of pseudotumor cerebri include;

a. papilledema.

b. severe headache

c. obesity.

d. visual field loss

Solution. c
Obesity is not an indication for treatment, although weight loss (even as little as 6 % of total body weight) often improves the condition.
Ref –Kanski6thedi. Pg 786

Correct Answer. c

(155). The most important predisposing factor for acute dacryocystitis is;

a. chronic blepharitis.

b. acute bacterial conjunctivitis

c. tear stasis of any etiology.

d. ethmoid sinusitis

Solution. c
Strictures, narrow or long ducts, and nasal and sinus inflammatory disease may cause tear stasis and lead to acute dacryocystitis.
Ref – Kanski6thedi. Pg 157

Correct Answer. c

Copyright © 2014 Delhi Academy of Medical Sciences, All Rights Reserved. 59/76
(156). Which of the following phakomatoses has/have an autosomal recessive inheritance pattern?

a. neurofibromatosis.

b. tuberous sclerosis

c. Sturge-Weber syndrome

d. ataxia-telangiectasia.

Solution. d
Five diseases characterized by disseminated hamartomas are known as phakomatoses – von Hippel – Lindau disease (angiomatosis
retinae),Sturge – Weber syndrome(encephalorigeminalangiomatosis), neurofibromatosis (von Recklinghausen’s disease), tuberous
sclerosis (Bourneville ‘s disease), and ataxia- telangiectasia (Louis – Bar syndrome). Of these, only ataxia – telangiectasia is inherited on
an autosomal recessive basis.Sturge – weber syndrome is sporadic The others are autosomal dominant.
Ref – Textbook of Clinical Ophthalmology 3th edi. Pg 431

Correct Answer. d

(157). With which of the following nerves does parasympathetic innervation to the iris sphincter travel?

a. nerve to the levator palpebrae superioris.

b. nerve to the inferior oblique.

c. nerve to the superior rectus.

d. nerve to the superior oblique.

Solution. b
Parasympatheticfiberstravel with the inferior division of the oculomotor nerve (CN111)
Ref – Wills eye Manual, 6thedi. Pg 394

Correct Answer. b

(158). Which one of the following is NOT involved with vertical eye movements?

a. Frontal eye fields

b. Paramediian pontine reticular formation

c. Interstitial nucleus of Cajal

d. Trochlear nucleus

Solution. b
The supra nuclear control of vertical saccades originates in the frontal eye fields or in the superior colliculus. They project to neutrons in
the rostral interstitial nucleus of. The medical longitudinal fascicles (riMLF) and on to the nuclei of cranial nerve III and IV. The
interstitial nucleus of Cajal is involved with vertical pursuit control. The Paramedian pontine reticular formation (PPRF) controls
horizontal eye movements.
Ref – Wills eye Manual, 6thedi. Pg 267

Correct Answer. b

Copyright © 2014 Delhi Academy of Medical Sciences, All Rights Reserved. 60/76
(159). 9 A patient has been on systemic corticosteroids for years. He notes blurred vision. Which of the following findings are most likely
related to the drug?

a. Anterior Subcapsular cataract.

b. Posterior Subcapsular cataract

c. Corneal edema need a period after edema

d. Iris neovascularization.

Solution. b
Long – term use of systemic corticosteroids often causes posterior subcapsular cataracts and many cause glaucoma.
Ref – Textbook of Clinical Ophthalmology 3th edi. Pg 84

Correct Answer. b

(160). What is true for Fuch’s dystrophy:

a. Characterized by presence of guttata in the cornea

b. Shows peripheral cornea guttata of Descemt’s membrane.

c. Has as its primary defect an epithelial abnormality.

d. Is most common in men.

Solution. a
Fuch’sdystrophy is bilateral, noninflammatory, progressive loss of endothelium, has excresences of descments membrane (guttat).It has
an autosomal dominant inheritance pattern, but is most commonly symptomatic in females. The key features are central
guttae,Descement’sfolds, stromal and microcystic epithelial edema. Endothelilaldysfunction resultsin corneal edema. Other features
include prominent erosions, and open – angle glaucoma.
Ref – Kanski6thedi. Pg 295

Correct Answer. a

(161). At presentation, a 48 – year-old patient has mildly descreased vision. Examination shows discrete corneal stromal opacities. The stroma
between the opacities is clear. The most likely diagnosis is:

a. Granular corneal dystrophy.

b. Macular corneal dystrophy.

c. Lattice dystrophy

d. Schnyder dystrophy

Solution. a
Granular stromal dystrophy is characterizedby crumb –like discrete opacities in the corneal stromawith clear interventing spaces. The
deposits consist of hyaline granules that stain red with Masson’s trichrome.
Ref – Kanski6thedi. Pg 292

Correct Answer. a

Copyright © 2014 Delhi Academy of Medical Sciences, All Rights Reserved. 61/76
(162). The following condition typically develops in all the following except:

a. Chronic tropical epinephrine therapy for glaucoma.

b. Uveitis.

c. Juvenile rheumatoid arthritis.

d. Hyperparathyroidism.

Solution. a
Preservatives in pilocarpine have been associated with band keratopathy. This disorder results from deposition of calcium at the level of
Bowman’smembran. It is seen most often in eyes with chronic disease such as uveitis, glaucoma, keratitis, and trauma. Systemic diseases
with elevated serum calcium can also result in band keratopathy.Treatmentis with disodium ethylenediaminetetraaceticacid(EDTA)
chelation or laser phototherapeutickeratectomy.
Ref – Wills eye Manual, 6th edi. Pg 394

Correct Answer. a

(163). Juvenile – onset glaucoma, false is:

a. Develops during the first – two decades of life.

b. May be caused by the gene product TIGR

c. May be caused by a gene located on chromosome Iq23.

d. Can be inherited as an autosomal recessive trait.

Solution. d
Juvenile – onsetglaucoma is inherited as an autosomal dominant trait.
Ref – Textbook of Clinical Ophthalmology 3th edi. Pg 70

Correct Answer. d

Copyright © 2014 Delhi Academy of Medical Sciences, All Rights Reserved. 62/76
(164). The following condition is seen in all the following except:

a. Soft contact lens wearers

b. Acrylic lens implant

c. Hard contact lens wearers

d. Ocular prostheses

Solution. b
Giant papillary conjunctivitis (GPC) is due to inflammation of the superior conjunctiva secondary to contact lens wear, ocular prosthesis,
or protruding ocular sutures. GPC is seen 10 minutes more frequently in soft contact lens wearers compared with hard contact lens
wearers. Symptoms include itching, mucoid dischargeand foreign body sensation. Treatment of the condition involves discontinuing until
offendingagent till symptoms improve. Mast cell stabilizers and a short – course of topical corticosteroids are usefulin severe cases.
Ref – Wills eye Manual, 6thedi. Pg 111

Correct Answer. b

(165). A 68 – years- old patient comes in with decreased vision of the right eye, correctable to 6/18 with refraction (-1. 75 sph). Two years ago
the refraction was -0. 50 sph. One year ago the refraction was - 1. 25 sph. The most likely diagnosis is:

a. Nuclear cataract.

b. Anterior cortical cataract.

c. Posteriorly subluxated lens.

d. Fuch’s corneal dystrophy.

Solution. a
Nuclear sclerosis of the lens often causes a slowly increasing myopia in patients.This can give rise to the“ secondsight” phenomena
where the patient is able to read without glasses.
Ref – Wills eye Manual, 6th edi. Pg 395

Correct Answer. a

Copyright © 2014 Delhi Academy of Medical Sciences, All Rights Reserved. 63/76
(166). Dorsal midbrain (Parinaud’s) syndrome consists of all except:

a. Light – near pupillary dissociation.

b. Lid retraction

c. Convergence – retraction nystagmus

d. downgaze paralysis

Solution. d
It causes upgaze palsy
Dorsal Midbrain Syndrome
Clinical features:
• Light-near-dissociation (response to accommodation better than to light)
• Lid retraction (Collier’s sign) • Supranuclear gaze palsy (normal vestibular
ocular reflex and Bell’s reflex) • Convergence retraction nystagmus • Spasm of convergence • Spasm of accommodation
Ref – Kanski6thedi. Pg 815

Correct Answer. d

(167). The pupillary defect that affects the afferent arm of the pupillary response is the:

a. Marcus Gunn pupil

b. Argyl Robberson pupil

c. Adies pupil

d. Horners pupil

Solution. a
A Marcus Gunn pupil is the classic afferent pupillary defect (APD) that we check with the swinging light test. The ArgylRobberson is the
syphilitic pupil that reacts with near vision, but doesn’t respond to light. Horners and Adies are disorders of the sympathetic and
parasympathetic efferent pupil response.
Ref – Wills eye Manual, 6thedi. Pg 245

Correct Answer. a

(168). Components of near reflex are all except

a. Convergence

b. Miosis

c. Anterior displacement of lens

d. Increased anterior curvature of lens

Solution. c
Anterior displacement of lens
Ref.: Read the text belowKhurana Anatomy and Physiology of eye 2nd edition pg 20
Sol :Changes in accommodation
1. slackening of zonules
2. increase ant curvature of lens
3. ant pole moves forward (not entire lens)
4. increased axial thickness
5. lens sinks down
6. Pupil constriction
7. Convergence
8. choroid stretched forwards
9. oraserrata moves forward
Ref – Wills eye Manual, 6th edi. Pg 394

Correct Answer. c

Copyright © 2014 Delhi Academy of Medical Sciences, All Rights Reserved. 64/76
(169). What age group has the highest prevalence of trachoma?

a. Infants

b. young children

c. teenagers

d. middle – aged adults

Solution. b
Children aged 2 to 9 years have the highest prevalence of trachoma.
Clinical features: The child presents with lot of itching and watering. There are follicles(boiled Sago grain
appearance) in the upper palpebral conjunctiva and upper limbus. Later they cicratisze into Arlts line and
Herberts follicles. Arlt’s line is a form of a conjuctivalscarwhich is linear in shape and appear to be in sulcus subtarsalis.
The follicles on bulbar conjunctiva is pathognomic
Ref – Kanski6thedi. Pg 230

Correct Answer. b

(170). Which of the following are considered indications for removal of an intraorbital foreign body except;

a. vegetable foreign body

b. any lead foreign body

c. anterior, easily approachable foreign bodies.

d. orbital apex location with good vision

Solution. b
Vegetable foreign bodies carry a high risk of infection. Lead can cause a granulomatousreaction, but only if far posterior should be left in
place. BB pellets(oftenmadefromlead)are commonly left within the orbit without consequence. Orbital apex foreign bodies are hazardous
to remove and should be pursued only if there is convincing evidence of direct optic nerve compromise (decreased vision, afferent
papillary defect (APD), dyschromatopsia).
Ref – Textbook of Clinical Ophthalmology 3th edi. Pg 249

Correct Answer. b

(171). The most common malignant neoplasm of the lacrimal gland is the;

a. pleomorphic adenoma

b. malignant mixed tumour

c. adenoid cystic carcinoma

d. mucoepidermoid carcinoma

Solution. c
Mucoepidermoid carcinomais distinctly uncommon.
Ref – Textbook of Clinical Ophthalmology 3th edi. Pg 350

Correct Answer. c

Copyright © 2014 Delhi Academy of Medical Sciences, All Rights Reserved. 65/76
(172). All of the following findings are consistent with an inferior orbital wall fracture and entrapment except;

a. infraorbital hypesthesia

b. enophthalmos.

c. subcutaneous emphysema

d. limitation in ocular motility in all fields of gaze

Solution. d
Orbital floor fractures usually produce verticallimitations of gaze. Globalmotility deficits generally indicate blunt trauma with muscleand
/ or nerve contusion.
Ref – Wills eye Manual, 6th edi. Pg 160

Correct Answer. b

(173). A 57-year-old man complains of flashing lights, floaters and a shade of darkness over his inferior nasal quadrant in one eye. Which is
likely diagnosis

a. Rhegmatogenous retinal detachment

b. Epi-retinal membrane

c. Vitreous hemorrhage

d. Macular hole

Solution. a
Schaffer’s sign is when you see pigment behind the lens on slit-lamp exam, and occurs with retinal detachments and the release of
pigment into the vitreous chamber. A macula-off retinal detachment is unfortunate, but isn’t an immediate emergency – it certainly needs
to be repaired, but can wait for a few days if necessary, as the damage to the detached macular photoreceptors has already occured. Epi-
retinal membranes are common and aren’t an emergency unless actively creating a tractional detachment. Vitreous hemorrhages are not
emergencies either, assuming there isn’t a detachment behind that blood on your ultrasound. Smaller retinal tears, however, need to be
treated early to make sure they don’t progress and peel off the macula.
Ref – Kanski6thedi. Pg 701

Correct Answer. a

(174). Which is not true for Congenital nasolacrimal obstruction:

a. Should usually treated by about age 4 year with irrigation and probing.

b. Should be treated with silicone intubation after two failed probing attempts

c. Should be treated with dacryocy storhinostomy if nasal probing cannot be performed.

d. Associated with amnioceles requires probing at an early age.

Solution. d
Congenitalamnioceles typically resolve without intervention, and therefore rarely require probing
Congenital nasolacrimalobstruction is characterised by:
1. Epiphora which develops after 7 days of birth followed by mucopurulent discharge
2. Regurgitation of mucopurulent discharge on applying pressure over lacrimal sac region.(positive regurgitation test)
3. Swelling over the sac area
Treatment:
1. Massage over thr lacrimal sac region with topical antibiotics cure the obstruction in approx 90% of the infants. Spontaneous
recanaliculization of an obstructed NLD can occur upto 9 months.
2. Probing of Nasolacrimal duct: it is performed if the condition is not cured by 1-12 months. Bowman’s probe is used for this purpose
3. Dacrocystorhinostomy: it is done at 4 years if the condition is still not resolved after repeated probing.
Ref – Kanski6thedi. Pg 158

Correct Answer. d

Copyright © 2014 Delhi Academy of Medical Sciences, All Rights Reserved. 66/76
(175). Glaucomatocyclitic crisis (Posner –Schlossman syndrome, what is true:

a. Mutton fat kps are seen

b. Closed angles on gonioscopy

c. Advanced visual field defect

d. Associated with high iop but few cells.

Solution. d
Anterior uveitis in the glaucomatocyclitic crises is significant for a markedabsence of cellular response.
Also called Hypertensive uveitis.
Clinical features:
1. IOP-: 40-60mmHg
2. Few depigmented KPs
3. Cornea is clear
4. AC has few cells
5. Gonioscopy shows open angles.
6. Optic nerve – No cupping
7. Visual field is normal
Ref – Wills eye Manual, 6th edi. Pg361

Correct Answer. d

(176). Which of the following extraocular muscles does not originate anatomically from the orbital apex?

a. superior rectus

b. superior oblique

c. inferior rectus.

d. inferior oblique

Solution. d
The inferior oblique is unique among the seven extraocular muscles in its anterior anatomic origin (from the orbital floor, just
posterolateral to the lacrimal sac fossa).
Ref – Wills eye Manual, 6th edi. Pg 153

Correct Answer. d

Copyright © 2014 Delhi Academy of Medical Sciences, All Rights Reserved. 67/76
(177). What procedure is being done

a. Sub tenon injection

b. Intravitreal injection

c. Retrobulbar injection

d. Peribulbar injection

Solution. b
Common Diseases Treated by Intravitreal Injections-
AMD (neovascular age related macular degeneration)
CSME/PDR (clinically significant macular edema/ proliferetive diabetic retinopathy)
Retinal Vein Occlusions
Endophthalmitis
Uveitis
CME (cystoid macular edema)
CNVM (choroidal neovascular membrane) secondary to multiple retinal diseases
Ref – Wills eye Manual, 6th edi. Pg 299

Correct Answer. b

(178). Which of the following is a poor prognostic sign in a patient with nonproliferative diabetic retinopathy?

a. numerous cotton – wool spots

b. numerous microaneurysms

c. extensive intraretinal microvascular abnormalities

d. extensive exudates

Solution. c
Formation of exentesiveintraretinal microvascular abnormalities implies widespread severe injury to small arterioles, with the resulting
ischemic state commonly being the immediate predecessor or neovascularization (NV).The other options are features of lower –risk
nonproliferative retinopathy, with the exception of disc neovascularzation (NVD) (which is a form of proliferative diabetic retinopathy).
Ref – Textbook of Clinical Ophthalmology 3th edi. Pg 223

Correct Answer. c

Copyright © 2014 Delhi Academy of Medical Sciences, All Rights Reserved. 68/76
(179). Retinal metastases are most commonly from a primary;

a. breast tumor

b. lung tumor

c. gastrointestinal tumor

d. cutaneous melanoma

Solution. d
Retinal metastases are rare. Cutaneous melanoma is the most common primary tumor.
Ref – Textbook of Clinical Ophthalmology 3th edi. Pg 348

Correct Answer. d

(180). Which of the following is / are considered defining features of the blepharophimosis syndrome?

a. ptosis.

b. epicanthus tarsalis

c. telecanthus.

d. lower lid ectropion

Solution. b
Blepharophimosis syndrome (congenital eyelid syndrome) is characterized by telecanthus; epicanthusinversus, severe ptosis, and
blepharophimosis.Ectropion is a variable feature.
Ref – Kanski 6thedi. Pg 143

Correct Answer. b

(181). Latent type 1 herpes simplex virus (HSV) (responsible for recurrent orofacial infection) generally resides in the;

a. oculomotor nucleus

b. gasserian ganglion

c. geniculate ganglion

d. sphenopalatine ganglion

Solution. b
Type 2 herpes usually resides latently in spinal ganglia. The gasserian (or trigeminal) ganglion, locatad in Meckel’s cave, contains the
cell bodies of te trigeminal nerve.
Ref – Textbook of Clinical Ophthalmology 3th edi. Pg 127

Correct Answer. b

(182). Surveilliance of the patient with Graves’ ophthalmopathy must include all except;

a. visual acuity testing

b. visual field testing

c. color vision testing

d. fluorescein angiography (FA).

Solution. d
Decreasingacuity, relative afferent papillary defect (APD), impairment of color vision, and evidence of scotomatamayall beindicativeof
optic nerve compression. Maculopathy is uncommonin Grave’s disease
Ref – Kanski6thedi. Pg 170

Correct Answer. d

Copyright © 2014 Delhi Academy of Medical Sciences, All Rights Reserved. 69/76
(183). Clinical findings that are more likely in pediatric orbital pseudotumor than in the adult variety of the disease include all of the following
except;

a. bilateral involvement

b. systemic symptoms and signs (malaise, fever, vomiting)

c. minimal periocular pain

d. peripheral eosinophilia.

Solution. c
Orbital pseudotumor is generallyquitepainful for bothchildren and adults.
Ref – Textbook of Clinical Ophthalmology 3th edi. Pg 507

Correct Answer. c

(184). Which of the following clinical features of newly diagnosed idiopathic pars planitis is most predictive of the future development of
multiple sclerosis (MS).

a. posterior subcapsular cataract.

b. 180 degrees of “snowbanking. “

c. retinal vascular sheathing.

d. angiographic optic disc edema.

Solution. c
The risk of future multiple sclerosis (MS) following a diagnosis of pars planitis exceeds 16 %. The risk was significantly greater among
patients presenting with retinal vascular sheathing.
Ref –Kanski6thedi. Pg 456

Correct Answer. c

(185). The finding that all three types of Duane’s syndrome share is;

a. a deficit of abduction.

b. a deficit of adduction.

c. globe retraction with adduction.

d. esotropia.

Solution. c
Duane’s syndrome may be secondary to hypoplasia or aplasia of the abducens nucleus with lateral rectus innervations by the oculomotor
nerve.It has various presentations, but retraction of the adducted globe appears most consistently. The three types of Duane’s syndrome
are distinguished by the relative ability to adduct or abduct; type 1, limited abduction but full adduction ; type 2, normal abduction but
limited adduction ; and type 3, both abduction and adduction are limited.
Ref – Textbook of Clinical Ophthalmology 3th edi. Pg 465

Correct Answer. c

Copyright © 2014 Delhi Academy of Medical Sciences, All Rights Reserved. 70/76
(186). A child with epicanthic folds, myopia, single transverse palmar crease and keratoconus presents to clinic. What anomaly (as seen in
picture) you are most likely to find?

a. corectopia

b. Lisch nodules

c. Koeppe nodule

d. Bruschfield spots

Solution. d
The clinical features described above are consistent with Down's syndrome (trisomy 21), which is associated with Bruschfield spots.
Ref – Wills eye Manual, 6th edi. Pg 394

Correct Answer. d

(187). A patient with a history of bilateral occipital lobe infarcts adamantly states that he can see quite well and confabulates visual images. He
most likely has;

a. Anton’s syndrome

b. palinopsia.

c. Charles Bonnet’s syndrome

d. blindsight.

Solution. a
This is a classic description of Anton’s syndrome.
Ref – Kanski6thedi. Pg 815

Correct Answer. a

(188). Relative to patients with primary open – angle glaucoma (POAG), which of the following conditions is associated with an equal or better
response to argon laser trabeculoplasty (ALT)

a. aphakic glaucoma

b. pigmentary glaucoma

c. uveitic glaucoma

d. angle closure glaucoma

Solution. b
Argon lasortrabeculoplasty(ALT) response appears to be better for pigmentary glaucoma and pseudoexfoliation and poorer for
inflammatory diseases, recessed angles, membranes in angles, young patients with developmental defects, and aphakic eyes.
Ref – Kanski6thedi. Pg 383

Correct Answer. b

Copyright © 2014 Delhi Academy of Medical Sciences, All Rights Reserved. 71/76
(189). The parents of a 2-year-old girl presents to your office pendular nystagmus and squints in bright light. The retina appears normal, but the
foveal reflex is blunted. Dark-adapted scotopic ERG responses are normal, but light-adapted photopic signals are greatly diminished. No
relatives are similarly affected. What condition does this patient most likely have?

a. Stargardt disease

b. congenital stationary night blindness

c. Leber congenital amaurosis

d. achromatopsia

Solution. d
The early onset of photophobia and nystagmus points to a congenital cone dysfunction.
The loss of photopic ERG response, including photopic flicker response, and the patient's essentially normal rod function confirm the
generalized cone abnormality. Although congenital stationary night blindness (CSNB) can reduce visual acuity and cause nystagmus, the
congenital loss of night vision from rod system abnormalities would cause an abnormal scotopic ERG response. CSNB is frequently an X-
linked recessive trait that affects males. In Leber congenital amaurosis, overall vision is very limited because of generalized retinal
dysfunction that causes loss of both rod and cone ERG responses. Stargardt disease often becomes evident during the grade school and
teenage years and typically shows a nearly normal rod and cone ERG response. Achromatopsia causes total color blindness as a result of
a congenital absence of cone photoreceptors. It has an autosomal recessive inheritance pattern, and its appearance in more than the
current generation is rare. Ultimate visual acuity ranges from 20/ 1 00 to 20/200 and is best in dimmer light or with sunglasses.
Ref - Alberts Principles and Practice of Ophthalmology, 3rdedi – pg 2253

Correct Answer. d

(190). Which upper-eyelid structure is considered to be analogous to the capsulopalpebral fascia of the lower eyelid?

a. Muller’s muscle

b. levator palpebrae superioris

c. levator aponeurosis

d. Whitnall’s ligament

Solution. c
The capsulopalpebralfascia is a condensation of noncontractile fascia from the sheaths of the inferior oblique and inferior rectusmuscle.
Like the aponeurosis of the levator muscle, it retracts the tarsus.
Ref – Wills eye Manual, 6th edi. Pg 145

Correct Answer. c

(191). Which of the following is most likely to be an extraocular focus of infection for endogenous bacterial endophthalmitis?

a. liver abscess

b. endocarditis.

c. orbital cellulitis

d. meningitis.

Solution. a
Liver abscess was the most common extraocular focusfor endogenous bacterial endophthalmitis, followed by endocarditis.
Ref – Kanski6thedi. Pg 483

Correct Answer. a

Copyright © 2014 Delhi Academy of Medical Sciences, All Rights Reserved. 72/76
(192). The key clinical feature distinguishing between nevus flammeus and capillary hemangioma is;

a. extent of skin thickening

b. lesion color

c. area of skin affected

d. presence or absence of blanching with ressure.

Solution. d
Capillary hemangiomata will blanch with pressure, whereasthe nevusflammeusdoesnot. Thereisconsiderable overlap between the
otherfeatures listed.
Ref – Kanski6thedi. Pg 186

Correct Answer. d

(193). Which is false for the following condition

a. Are usually present at birth

b. Regress by age 7 years in 75 % of affected individuals.

c. May be associated with the Kasabach – Merritt syndrome

d. Affect girls more frequently than boys

Solution. a
The capillary hemangioma is a common vascular lesion of childhood and is the most common orbital found in children. Usually the
lesions manifest by 6 months of age, with only one – third of lesions visible at birth. Patients who have extensive visceral hemangiomas
may develop thrombocytopenia reltated to entrapment of platelets within the lesion, with resulting hemorrhagic diathesis, known as the
kasabach – Merritt syndrome.
Ref – Kanski6thedi. Pg 186

Correct Answer. a

Copyright © 2014 Delhi Academy of Medical Sciences, All Rights Reserved. 73/76
(194). Which of the following regarding dermoid and epidermoid cysts is false?

a. They share a common pathophysiology.

b. The key distinguishing feature between the two is the nature of the wall of the cystic cavity.

c. Superficial cysts present more often during childhood.

d. In adults, nearly all of these lesions are posterior to the orbital septum.

Solution. d
Dermoid cysts are usually congenital (a type of choristomaie. lesion containingmature tissue but found at an abnormal location). They
may be found in the eyelid, peripheral cornea or orbit. In the latter, the most common location is in the superotemporal quadrant. The
content of the wall comes from desquamation of the epithelium and secretion of the sebaceous glands lining the wall.
Epidermoid cysts come from squamous epithelial cells that have penetrated deep into the dermisfor example after skin surgery, trauma
or congenital. The content of the cyst is mainly keratin as only the cysts do not contain dermal structures such as hair follicle or
sebaceous glands.
Histologically, the two differs in the structures found in the lining of the wall.
Features common to both are:
keratinized and stratified squamous epithelium, eosinophilic (pinkish when stained with H&E) content within the wall. However, in
dermoid but not in epidermoid the following are present:
presence of sebaceous glands, sweat glands and hair follicles ie. structures fromthe dermis.
Ref – Kanski6thedi. Pg 185

Correct Answer. d

(195). Which one of the following characteristics is felt to confer the greatest protection from the development of proliferative diabetic
retinopathy?

a. complete posterior vitreous separation (PVS)

b. partial PVS

c. younger age (< 30 years)

d. ipsilateral carotid artery stenosis

Solution. a
The vitreous acts as a scaffold for the growth of neovascularization (NV) and will exert traction once it has developed, leading to
hemorrhage and retinal detachment (RD)
Ref – Kanski6thedi. Pg 566

Correct Answer. a

(196). In retinoblastoma, which is true

a. Calcification is an important diagnostic feature

b. Rosettes indicate a poorer prognosis

c. Diffuse infiltrative retinoblastoma is usually seen in younger children

d. Haematogenous spread is more common

Solution. a
The intraocular calcification is uncommon in any other childhood condition. Rosettes indicate partial differentiation towards
photoreceptors.
Ref – Kanski6thedi. Pg 544

Correct Answer. a

Copyright © 2014 Delhi Academy of Medical Sciences, All Rights Reserved. 74/76
(197). Regarding ophthalmic artery, false among following is:

a. Enters the optic nerve 8 to 12 m behind the globe.

b. is 2nd branch of Internal carotid artery

c. Is renamed the central retinal artery within the globe.

d. supplies retina and choroid

Solution. b
The ophthalmic artery (OA) is the first branch of the internal carotid artery distal to the cavernous sinus. Branches of the OA supply all
the structures in the orbit as well as some structures in the nose, face and meninges The OA emerges from the internal carotid artery
usually just after the latter emerges from the cavernous sinus although in some cases, the OA branches just before the internal carotid
exits the cavernous sinus. The OA arises from the internal carotid along the medial side of the anterior clinoid process and runs
anteriorly passing through the optic canal with and inferolaterally to the optic nerve. The ophthalmic artery can also pass superiorly to
the optic nerve in a minority of cases. In the posterior third of the cone of the orbit, the ophthalmic artery turns sharply medially to run
along the medial wall of the orbit.
Ref – Wills eye Manual, 6th edi. Pg 300

Correct Answer. b

(198). If a patient has dense white cataract and the posterior pole is not visible, which of the following would be most helpful for the clinician to
know about posterior segment status

a. Amsler grid

b. Ultrasound B scan

c. Ultrasound A scan

d. Fluorescein angiography

Solution. b
B-scan ultrasonography is indicated to evaluate for occult tumors, retinal detachment, or posterior staphyloma or other posterior
pathology that could affect visual outcome. Laser interferometry, Maddox rod testing, and photostress recovery are not reliable with
such a dense cataract. Specular microscopy would be indicated if signs of corneal endothelial dysfunction were present.
Ref – Kanski6thedi. Pg 44

Correct Answer. b

(199). Most common cause of spontaneous vitreous hemorrage in an adult is

a. Diabetic retinopathy- non proliferative

b. Proliferative diabetic retinopathy

c. Age related macular degeneration

d. Posterior vitreous detachment

Solution. b
Complications of PDR:
1. Presistent intra-gel vitreous haemorrhage
2. RD: Tractional RD can later lead to Rhegmatogenous RD.
3. Opaque membranes- on posterior surface of detached hyaloid
4. Burnt-out stage: Increase in fibrous component
5. Rubeosisiridis: NVG
Ref – Kanski6thedi. Pg 567

Correct Answer. b

Copyright © 2014 Delhi Academy of Medical Sciences, All Rights Reserved. 75/76
(200). False for Neuroretinitis:

a. is associated with macular star sign

b. Can be associated with cat – scratch fever

c. Causes optic disk edema and macular exudation.

d. is associated with demyelination

Solution. d
Neuroretinitis is characterized by an inflammation of the optic disc vasculature with exudation of fluid into the peripapillary retina.
Ref – Wills eye Manual, 6th edi. Pg 248

Correct Answer. d

Test Answer
1.(d) 2.(c) 3.(d) 4.(a) 5.(d) 6.(a) 7.(c) 8.(d) 9.(b) 10.(b)

11.(d) 12.(c) 13.(a) 14.(b) 15.(c) 16.(d) 17.(d) 18.(c) 19.(a) 20.(a)

21.(b) 22.(a) 23.(b) 24.(d) 25.(c) 26.(d) 27.(d) 28.(b) 29.(c) 30.(a)

31.(a) 32.(c) 33.(a) 34.(c) 35.(c) 36.(c) 37.(d) 38.(d) 39.(a) 40.(b)

41.(c) 42.(b) 43.(c) 44.(d) 45.(a) 46.(b) 47.(d) 48.(c) 49.(b) 50.(d)

51.(d) 52.(a) 53.(b) 54.(c) 55.(c) 56.(a) 57.(b) 58.(d) 59.(d) 60.(b)

61.(a) 62.(c) 63.(d) 64.(d) 65.(b) 66.(c) 67.(c) 68.(d) 69.(a) 70.(c)

71.(b) 72.(d) 73.(a) 74.(d) 75.(b) 76.(c) 77.(d) 78.(d) 79.(d) 80.(c)

81.(b) 82.(a) 83.(c) 84.(b) 85.(c) 86.(b) 87.(d) 88.(d) 89.(d) 90.(d)

91.(c) 92.(a) 93.(d) 94.(a) 95.(b) 96.(b) 97.(a) 98.(c) 99.(d) 100.(a)

101.(d) 102.(d) 103.(c) 104.(b) 105.(b) 106.(d) 107.(d) 108.(d) 109.(c) 110.(d)

111.(d) 112.(d) 113.(b) 114.(d) 115.(d) 116.(b) 117.(c) 118.(b) 119.(d) 120.(d)

121.(c) 122.(c) 123.(c) 124.(d) 125.(c) 126.(d) 127.(a) 128.(d) 129.(d) 130.(b)

131.(b) 132.(a) 133.(c) 134.(c) 135.(b) 136.(d) 137.(a) 138.(c) 139.(c) 140.(a)

141.(a) 142.(d) 143.(a) 144.(b) 145.(d) 146.(c) 147.(a) 148.(a) 149.(b) 150.(a)

151.(b) 152.(d) 153.(b) 154.(c) 155.(c) 156.(d) 157.(b) 158.(b) 159.(b) 160.(a)

161.(a) 162.(a) 163.(d) 164.(b) 165.(a) 166.(d) 167.(a) 168.(c) 169.(b) 170.(b)

171.(c) 172.(b) 173.(a) 174.(d) 175.(d) 176.(d) 177.(b) 178.(c) 179.(d) 180.(b)

181.(b) 182.(d) 183.(c) 184.(c) 185.(c) 186.(d) 187.(a) 188.(b) 189.(d) 190.(c)

191.(a) 192.(d) 193.(a) 194.(d) 195.(a) 196.(a) 197.(b) 198.(b) 199.(b) 200.(d)

Copyright © 2014 Delhi Academy of Medical Sciences, All Rights Reserved. 76/76

Das könnte Ihnen auch gefallen